You are on page 1of 201

^

N a tio n a l B o a rd o f M e d ic a l E xa m in e rs - G o o g le C h ro m e

i m

. _______________

['

cj

| 0

https://ww w .starttest.eom /api/5.l.1.0/TTDStart.aspx?SVC=lbfd4732-460a-454b-90b5-9985c0560007

Exam Section 1: Item 1 o f 50 H Mark

National Board o f Medical Examiners Comprehensive Basic Science Self-Assessment

Tim e Remaining: 3 hr 59 min 9 sec

1. A 62-year-old man is brought to the physician by his wife because of increasing confusion during the past 6 months. His wife says that he has become lost twice in the past month when going to work, even though he has been going to the same office for 12 years. She adds that he often has difficulty finding objects such as his glasses and keys, sometimes cannot recall his grandchildren's names, and has become very critical of her cooking, which he used to enjoy. When asked, he can name only the current president and none of the candidates for an upcoming presidential election, although he and his wife watch the television news together each night. Neurologic examination shows no motor or sensory abnormalities. His Mini-Mental State Examination score is 19/30. Treatment with a cholinesterase inhibitor is most likely to improve this patient's memory because of its ability to target synaptic connections between which of the following structures? o A) The basal forebrain and neurons in the cerebral cortex B) The dentate nucleus and the thalamus C) The fornix and neurons in the mammillary bodies D) The substantia nigra and the neurons of the globus pallidus O E) The thalamus and neurons in layer 4 of the cerebral cortex

https://www.starttest.com/api/5.1.1.0/TTDStart.aspx?SVC=lbfd4732-460a-454b-90b5-9985c0560007

Exam Section 1: Item 2 o f 50 H Mark

National Board o f Medical Examiners Comprehensive Basic Science Self-Assessment

Tim e Remaining: 3 hr 58 min 52 sec

2. A 25-year-old man has acute lymphocytic leukemia that improves after the administration of chemotherapy including methotrexate. The beneficial effect of this drug is most likely the result of inhibition of which of the following enzyme activities? A) C) Adenosine deaminase Hypoxanthine-guanine phosphoribosyltransferase

o B) DNA polymerase

0D) Thymidylate synthase


C E) Xanthine oxidase

N a tio n a l B o a rd o f M e d ic a l E xa m in e rs - G o o g le C h ro m e

i m

. _______________

['

cj

| 0

Q https://www.starttest.eom/api/5.l.1.0/TTDStart.aspx?SVC=lbfd4732-460a-454b-90b5-9985c0560007 Exam Section 1: Item 3 o f 50 H Mark National Board o f Medical Examiners Comprehensive Basic Science Self-Assessment Tim e Remaining: 3 hr 58 min 43 sec

3. A 30-year-old woman comes to the physician for a follow-up examination because of a 10-year history of recurrent renal calculi. The calculi have primarily been composed of calcium oxalate. Physical examination shows no abnormalities. Serum calcium and phosphorus concentrations are within the reference ranges. A 24-hour urine collection shows an increased calcium concentration. The physician recommends that the patient increase her consumption of water and decrease her consumption of sodium and protein. The physician also recommends pharmacotherapy with a drug that decreases urinary calcium concentrations. Which of the following is the most likely mechanism of action of this drug? A) Decreases 1,25-dihydroxycholecalciferol synthesis B) Decreases H+secretion in the distal tubule C) Decreases magnesium reabsorption in the thick ascending limb of the loop of Henle o D) Increases calcium reabsorption in the distal tubule E) Increases H C 0 3 - secretion in the proximal tubule F) Increases phosphate reabsorption in the proximal tubule G) Inhibits parathyroid hormone secretion

https://www.starttest.com/api/5.1.1.0/TTDStart.aspx?SVC=lbfd4732-460a-454b-90b5-9985c0560007

Exam Section 1: Item 4 o f 50 H Mark

National Board o f Medical Examiners Comprehensive Basic Science Self-Assessment

Tim e Remaining:

4. A 55-year-old woman undergoing chemotherapy for melanoma develops a temperature of 39C (102.2F) and neutropenia. Her absolute segmented neutrophil count is 500/mm3. In addition to antibiotics, which of the following substances is most appropriate to administer to this patient? o A) Granulocyte-macrophage colony-stimulating factor B) lnterleukin-8 (IL-8) CJIL-10 D) Macrophage colony stimulating-factor E) Stem cell factor (C-kit ligand)

LI

https://www.starttest.com/api/5.1.1.0/TTDStart.aspx?SVC=lbfd4732-460a-454b-90b5-9985c0560007

Exam Section 1: Item 5 o f 50 H Mark

National Board o f Medical Examiners Comprehensive Basic Science Self-Assessment

Tim e Remaining: 3 hr 58 min 21 sec

5. A 40-year-old woman comes to the physician because of a 2-month history of increasing redness around her nose and cheeks, and pimples around her mouth. She appears very upset and tells the physician, "I look like a teenaged Santa Claus." Which of the following initial responses by the physician is most appropriate? O A) "Can you tell me how often you go out in the sun and what kind of protection you use?" O B) "Don't worry. Let me assure you this is not a serious condition." o C) "It's upsetting when you have a skin problem on your face." D) "Maybe a little. At least you are keeping your sense of humor about the situation." O E) "You really do look rather odd. I can understand why you are so upset."

LI

N a tio n a l B o a rd o f M e d ic a l E xa m in e rs - G o o g le C h ro m e

i m

. _______________

['

cj

| 0

Q https://www.starttest.eom/api/5.l.1.0/TTDStart.aspx?SVC=lbfd4732-460a-454b-90b5-9985c0560007 Exam Section 1: Item 6 o f 50 H Mark National Board o f Medical Examiners Comprehensive Basic Science Self-Assessment Tim e Remaining:

6. A 46-year-old man is brought to the emergency department 2 hours after he fell headfirst into the water while attempting to water ski. He says that he has pain when he tries to move his left arm. Physical examination shows that the patient is unable to abduct the left upper extremity due to pain. Which of the following muscles is most likely injured in this patient? A) Infraspinatus B) Subscapularis o C) Supraspinatus D) Teres major E) Teres minor

https://www.starttest.com/api/5.1.1.0/TTDStart.aspx?SVC=lbfd4732-460a-454b-90b5-9985c0560007

Exam Section 1: Item 7 o f 50 0 Mark

National Board o f Medical Examiners Comprehensive Basic Science Self-Assessment

Tim e Remaining: 3 hr 58 min 11 sec

7. A 6-year-old girl is brought to the physician by her mother because of a 2-week history of increased thirst and a 3-kg (6.6-lb) weight loss. Her mother says that the patient is constantly drinking water. She is at the 75th percentile for height and 50th percentile for weight. Physical examination shows tachypnea and dehydration. Laboratory studies show hyperglycemia, metabolic acidosis, and ketonemia. If a biopsy specimen were obtained from this patient's pancreas, which of the following findings in islet cells would now be most likely? A) C) Basement membrane thickening of capillaries Decrease in mass and deposition of amyloid

o B) Cellular necrosis and lymphocytic infiltration O D) Large (3 cells and nuclei E) Marked atrophy and fibrosis

LI

N a tio n a l B o a rd o f M e d ic a l E xa m in e rs - G o o g le C h ro m e

i m

. _______________

['

cj

| 0

Q https://www.starttest.eom/api/5.l.1.0/TTDStart.aspx?SVC=lbfd4732-460a-454b-90b5-9985c0560007 Exam Section 1: Item 8 o f 50 H Mark National Board o f Medical Examiners Comprehensive Basic Science Self-Assessment Tim e Remaining: 3 hr 52 min 23 sec

8. A 1-year-old girl is brought to the emergency department because of a 5-day history of vomiting, copious diarrhea, and markedly decreased urinary frequency. Several other children at the day-care center that she attends have had a similar illness during the past week. She appears listless. Physical examination shows decreased skin turgor, cracked lips, and a dry mouth. The virus identified by ELISA testing of the stool is resistant to treatment with ether and contains a double-stranded, segmented RNA genome. Which of the following is the most likely causal virus? O A) Adenovirus B) Cytomegalovirus O C) Hepatitis A virus O D) Herpes simplex virus 1 O E) Influenza virus o F) Rotavirus

N a tio n a l B o a rd o f M e d ic a l E xa m in e rs - G o o g le C h ro m e

i m

. _______________

['

cj

| 0

Q https://www.starttest.eom/api/5.l.1.0/TTDStart.aspx?SVC=lbfd4732-460a-454b-90b5-9985c0560007 Exam Section 1: Item 9 o f 50 H Mark National Board o f Medical Examiners Comprehensive Basic Science Self-Assessment Tim e Remaining: 3 hr 52 min 19 sec

9. A 28-year-old woman, gravida 4, para 0, aborta 3, with systemic lupus erythematosus comes to the physician at 12 weeks' gestation for an initial prenatal examination. She is concerned that she will not be able to carry her current pregnancy to term. The presence of which of the following is most likely increasing the risk for complications during pregnancy in this patient? A) Antimitochondrial antibodies B) Anti neutrophil cytoplasmic antibodies o C) Antiphospholipid antibodies D) Antistreptolysin 0 antibodies O E) Rheumatoid factor

N a tio n a l B o a rd o f M e d ic a l E xa m in e rs - G o o g le C h ro m e

i m

. _______________

['

cj

| 0

Q https://www.starttest.eom/api/5.l.1.0/TTDStart.aspx?SVC=lbfd4732-460a-454b-90b5-9985c0560007 Exam Section 1: Item 10 o f 50 H Mark National Board o f Medical Examiners Comprehensive Basic Science Self-Assessment Tim e Remaining: 3 hr 51 min 28 sec

10. A study is conducted to assess the accuracy of a new rapid test to detect a virulent bacterial infection. This infection has an 80% mortality rate if it is not identified early in its course; however, prompt administration of antibiotics decreases the mortality rate to less than 5%. The risks of this antibiotic therapy are minimal. A total of 10,000 participants are enrolled and undergo assessment with the new test. The graph shows the distribution of infected and noninfected participants according to the results of the test. Which of the following labeled points is most appropriate for use as the optimal diagnostic cut point for results of this test?

A)

m B)

C)

D)

E)

https://www.starttest.com/api/5.1.1.0/TTDStart.aspx?SVC=lbfd4732-460a-454b-90b5-9985c0560007

Exam Section 1: Item 11 o f 50 H Mark

National Board o f Medical Examiners Comprehensive Basic Science Self-Assessment

Tim e Remaining: 3 hr 49 min 0 sec

11. A 60-year-old man comes to the physician because of a 6-month history of decreased libido and inability to sustain an erection. He also has had decreased energy during this period and has been falling asleep by 7 pm each evening. He has not had chronic pain, feelings of depression, or recent psychosocial stressors. He has had a 2.54-cm (1 -in) loss in height and a 7.62-cm (3-in) increase in waist circumference since his last visit 6 months ago. Physical examination shows mild gynecomastia and decreased muscle mass throughout. Which of the following is the most appropriate pharmacotherapy for this patient? A) Epoetin alfa

0 B) Human growth hormone C) Modafinil


D) Sildenafil o E) Testosterone

LI

https://www.starttest.com/api/5.1.1.0/TTDStart.aspx?SVC=lbfd4732-460a-454b-90b5-9985c0560007

Exam Section 1: Item 12 o f 50 H Mark

National Board o f Medical Examiners Comprehensive Basic Science Self-Assessment

Tim e Remaining: 3 hr 46 min 22 sec

12.

An 80-year-old woman is brought to the emergency department by police 30 minutes after she was found wandering in the street, confused and disoriented. She says that she has no medical conditions and takes no medications. During history taking, she becomes annoyed and agitated, saying, "Stop being so nosey! There's nothing wrong with me! I feel fine and I just want to go home." She appears frail but alert. Her vital signs are within normal limits. Physical examination shows mildly dry mucous membranes. There is no evidence of trauma. She is oriented to person but not to place or time. She is able to provide her home address. Which of the following is the most appropriate action by the physician?

o A) Attempt to contact the patient's family to find out more information B) Arrange for transportation to take the patient home C) Arrange for the patient to be transferred to a skilled nursing care facility D) Administer a sedative-hypnotic medication E) Admit the patient to a psychiatric inpatient facility

N a tio n a l B o a rd o f M e d ic a l E xa m in e rs - G o o g le C h ro m e

i m

. _______________

['

cj

| 0

Q https://www.starttest.eom/api/5.l.1.0/TTDStart.aspx?SVC=lbfd4732-460a-454b-90b5-9985c0560007 Exam Section 1: Item 13 o f 50 H Mark National Board o f Medical Examiners Comprehensive Basic Science Self-Assessment Tim e Remaining: 3 hr 42 min 55 sec

13. A 25-year-old man comes to the emergency department 5 hours after developing shortness of breath and chest pain during exercise; he has had no cough or bloody mucus. He has asthma and major depressive disorder. Current medications include fluticasone inhaler and albuterol. His temperature is 37.1C (98.8F), pulse is 110/min, respirations are 30/min, and blood pressure is 90/60 mm Hg. Pulse oximetry on room air shows an oxygen saturation of 93%. Cardiac examination shows a normal and S 2with no murmurs and no increase in jugular venous pressure. Laboratory studies show: Hemoglobin Hematocrit 13 g/dL 39%

National Board of Medical Examiners - Google Chrom e Q https:/7www.starttest.com/api/5.1.1.0/ITDStart.aspx?SVC=lbfd4732-460a-454b-90b5-9985c0560007

Exam Section 1: Item 13 o f 50 I Mark

National Board o f Medical Examiners Comprehensive Basic Science Self-Assessment

Tim e Remaining: 3 hr 42 min 44 sec

13. A 25-year-old man comes to the emergency department 5 hours after developing shortness of breath and chest pain during exercise; he has had no cough or bloody mucus. He has asthma and major depressive disorder. Current medications include fluticasone inhaler and albuterol. His temperature is 37.1C (98.8F), pulse is 110/min, respirations are 30/min, and blood pressure is 90/60 mm Hg. Pulse oximetry on room air shows an oxygen saturation of 93%. Cardiac examination shows a normal and S 2with no murmurs and no increase in jugular venous pressure. Laboratory studies show: Hemoglobin Hematocrit Arterial blood gas analysis on room air pH Pco2 Po2 A chest x-ray is shown. Which of the following pulmonary findings are most likely in this patient? A) Crackles on the left lung base and apex B) Crackles on the right lung base o C) Decreased breath sounds on the left C D) Increased wheezes on the left C E) Rhonchi on the right 13 g/dL 39% 7.46 26 mm Hg 60 mm Hg

https://www.starttest.com/api/5.1.1.0/TTDStart.aspx?SVC=lbfd4732-460a-454b-90b5-9985c0560007

Exam Section 1: Item 14 o f 50 0 Mark

National Board o f Medical Examiners Comprehensive Basic Science Self-Assessment

Tim e Remaining: 3 hr 40 min 56 sec

14.

A 45-year-old man has a partial colectomy for carcinoma. Which of the following scenarios indicates the highest likelihood for survival for 5 years after resection of the lesion? A) Moderately differentiated carcinoma invading the muscularis B) Mucin-producing carcinoma invasive to the serosal surface C) Mucin-producing carcinoma metastatic to two regional lymph nodes

o D) Poorly differentiated carcinoma confined to the mucosa E) Well-differentiated carcinoma with hepatic metastasis

LI

^ Q

National

- Google Chrom e

https://www.starttest.com/api/5.1.1.0/ITDStart.aspx?SVC=lbfd4732-460a-454b-90b5-9985c0560007

Exam Section 1: Item 15 o f 50 Mark

National Board o f Medical Examiners Comprehensive Basic Science Self-Assessment

Tim e Remaining: 3 hr 39 min 3 sec

15. A 28-year-old woman undergoes PPD skin testing to determine previous infection with Mycobacterium tuberculosis. Results show a 25-mm, firm area of induration at 48 hours. Analysis of this patient's lesion is most likely to show a predominance of which of the following cell types? O A) B lymphocytes o B) Cytotoxic T lymphocytes O C) Eosinophils ^ D) Macrophages E) Mast cells O F) Neutrophils

Previous

Lab Values

Review

Help -

Pause

O
..in

< >

1:44 PM 04/08/2012

N a tio n a l B o a rd o f M e d ic a l E xa m in e rs - G o o g le C h ro m e

i m

. _______________

['

cj

| 0

Q https://www.starttest.eom/api/5.l.1.0/TTDStart.aspx?SVC=lbfd4732-460a-454b-90b5-9985c0560007 Exam Section 1: Item 16 o f 50 H Mark National Board o f Medical Examiners Comprehensive Basic Science Self-Assessment Tim e Remaining: 3 hr 36 min 44 sec

16. A 37-year-old man comes to the emergency department because of a 10-day history of yellow skin, vague abdominal discomfort, and fatigue. His vital signs are within normal limits. Physical examination shows generalized icterus. Serum studies show: AST ALT Hepatitis B surface antigen (HBsAg) IgM anti-hepatitis B core antigen (HBcAg) Anti-hepatitis D virus 320 U/L 340 U/L positive positive negative

Direct damage to infected hepatocytes in this patient is most likely mediated predominantly by which of the following? O A) Antibody against HBcAg B) Antibody against HBsAg C) Complement via the alternate pathway D) Cytotoxic T lymphocytes o E) Viral cytopathic effect

National Board of Medical Examiners - Google Chrom e

Q https://wvvw.starttest.com/api/5.1.1.0/ITDStart.aspx?SVC=lbfd4732-460a-454b-90b5-9985c0560007

Exam Section 1: Item 17 o f 50 H Mark

National Board o f Medical Examiners Comprehensive Basic Science Self-Assessment

Tim e Remaining: 3 hr 35 min 46 sec

17. A defect in enzyme activity in amino acid metabolism is induced in an experimental animal. Results show an increased serum concentration of phenylalanine and decreased concentrations of dopamine and serotonin in the brain. A deficiency of which of the following is the most likely cause of the findings in this animal? O A) Biotin O B) Glutamate C) NADH o D) Tetrahydrobiopterin E) Vitamin B 12(cobalamin)

Previous

Lab Values

Review

Help -

Pause

O
..in

< >

1:47 PM 04/08/2012

National Board of Medical Examiners - Google Chrom e Q https:/7www.starttest.com/api/5.1.1.0/ITDStart.aspx?SVC=lbfd4732-460a-454b-90b5-9985c0560007

Exam Section 1: Item 18 o f 50 I Mark

National Board o f Medical Examiners Comprehensive Basic Science Self-Assessment

Tim e Remaining: 3 hr 35 min 1 sec

18. A 22-year-old man comes to the physician 1 week after he noticed a painless ulcer on his penis. Three weeks ago, he had sexual intercourse with a new male partner. Physical examination shows regional lymphadenopathy. A photograph of the penis is shown. Dark-field microscopy of exudate from the ulcer shows a spiral-shaped organism. Which of the following is the most appropriate pharmacotherapy for this patient? C A) Cefazolin C B) Ciprofloxacin C C) Erythromycin o D) Penicillin E) Trimethoprim-sulfamethoxazole

https://www.starttest.com/api/5.1.1.0/TTDStart.aspx?SVC=lbfd4732-460a-454b-90b5-9985c0560007

Exam Section 1: Item 19 o f 50 0 Mark

National Board o f Medical Examiners Comprehensive Basic Science Self-Assessment

Tim e Remaining: 3 hr 32 min 35 sec

19. A 23-year-old woman comes to the physician for prenatal care. On examination, she appears euthyroid and the thyroid gland is not palpable. Uterine size is consistent with dates, and fetal pulse is normal. Serum thyroid tests show: Thyroxine (T4) Thyroid-stimulating hormone 14 pg/dL 2 [jU/mL

Changes in which of the following hormones during pregnancy is primarily responsible for the thyroid test results? O A) Androgens O B) Corticosteroids o C) Estrogens O D) Progesterones 0 E) Prolactins

LI

N a tio n a l

Beard of Medical Examiners - Google Chrome

Q https://www.starttest.com/api/5.1.1.0/ITDStart.aspx?SVC=lbfd4732-460a-454b-90b5-9985c0560007 Exam Section 1: Item 20 o f 50 Q Mark National Board o f Medical Examiners Comprehensive Basic Science Self-Assessment Tim e Remaining: 3 hr 29 min 27 sec

20. A 56-year-old woman has recently diagnosed carcinoma of the breast. An x-ray of the chest shows a tumor next to the right side of the heart. An enhanced CT scan with the tumor invading the pericardium is shown. Which of the following structures is most likely involved? O A) Coronary sinus B) Greater splanchnic vein o C) Right phrenic nerve O D) Right vagus nerve E) Thoracic duct

https://www.starttest.com/api/5.1.1.0/TTDStart.aspx?SVC=lbfd4732-460a-454b-90b5-9985c0560007

Exam Section 1: Item 21 o f 50 H Mark

National Board o f Medical Examiners Comprehensive Basic Science Self-Assessment

Tim e Remaining: 3 hr 27 min 25 sec

21.

An 18-month-old girl is brought to the physician by her parents because of physical and mental regression. Pregnancy and delivery were normal. One month after birth, the mother noticed a marked startle reaction to noise. At the age of 1 year, she was not able to sit or stand. Physical examination shows macrocephaly. Ophthalmologic examination shows a cherry-red spot in the macula. Laboratory studies in blood leukocytes show a deficiency of (3-hexosaminidase A activity. An excessive amount of which of the following metabolites is most likely present in this patient's tissues and organs?

O A) Cholesterol B) Dermatan sulfate O C) Glycogen o D) GM2ganglioside E) Sphingomyelin

N a tio n a l B o a rd o f M e d ic a l E xa m in e rs - G o o g le C h ro m e

i m

. _______________

['

cj

| 0

Q https://www.starttest.eom/api/5.l.1.0/TTDStart.aspx?SVC=lbfd4732-460a-454b-90b5-9985c0560007 Exam Section 1: Item 22 o f 50 H Mark National Board o f Medical Examiners Comprehensive Basic Science Self-Assessment Tim e Remaining: 3 hr 26 min 6 sec

22. A 40-year-old woman with chronic paraplegia caused by multiple sclerosis is brought to the physician because of severe, debilitating muscle cramps for the past 3 weeks. Treatment with baclofen resolves her muscle cramps. Which of the following receptors most likely decreased the muscle spasticity in this patient? O A) a2 -Adrenoreceptor O B) (32 -Adrenoreceptor o C) Y-Aminobutyric acid Breceptor O D) Calcium-sensing receptor O E) Ryanodine receptor

N a tio n a l B o a rd o f M e d ic a l E xa m in e rs - G o o g le C h ro m e

i m

. _______________

['

cj

| 0

Q https://www.starttest.eom/api/5.l.1.0/TTDStart.aspx?SVC=lbfd4732-460a-454b-90b5-9985c0560007 Exam Section 1: Item 23 o f 50 H Mark National Board o f Medical Examiners Comprehensive Basic Science Self-Assessment Tim e Remaining: 3 hr 25 min 24 sec

23.

Nitrogen dioxide inhalation in humans causes loss of type I pneumocytes in those alveoli exposed to high concentrations of the gas. Within 2 weeks, the affected alveoli are reepithelialized with type I pneumocytes. Which of the following is the most likely immediate precursor of the new type I pneumocytes? A) Alveolar macrophages B) Capillary endothelial cells C) Interstitial cells

O D) Type I pneumocytes o E) Type II pneumocytes

LI

https://www.starttest.com/api/5.1.1.0/TTDStart.aspx?SVC=lbfd4732-460a-454b-90b5-9985c0560007

Exam Section 1: Item 24 o f 50 H Mark

National Board o f Medical Examiners Comprehensive Basic Science Self-Assessment

Tim e Remaining: 3 hr 24 min 23 sec

24. A 72-year-old man has had moderate progressive dementia for the past 3 years. He has a 25-year history of hypertension. He has a 5-year history of episodic dysarthria and weakness of the right upper extremity. He has no family history of a similar disorder. Which of the following is the most likely diagnosis? O A) AIDS dementia O B) Dementia, Alzheimer type O C) Huntington disease o D) Multi-infarct (vascular) dementia O E) Parkinsonism

National Board of Medical Examiners - Google Chrom e

Q h ttps://w w w .starttest.com /api/5.1 .1.0/TTDStart.aspx?SVC=lbfd4732-460a-454b-90b5-9985c0560007

Exam Section 1: Item 25 o f 50 H Mark

National Board o f Medical Examiners Comprehensive Basic Science Self-Assessment

Tim e Remaining: 3 hr 22 min 16 sec

25. A 56-year-old man is brought to the emergency department 1 hour after the sudden onset of left-sided weakness. Neurologic examination shows weakness of the lower two thirds of the face on the left, marked weakness of the left upper extremity, and moderate weakness of the left lower extremity. Deep tendon reflexes are hyperactive in both the upper and lower extremities on the left, but they are more active in the upper extremity. Normal results of MR angiography are shown. This patient most likely has an occlusion of which of the following labeled arteries?

Previous

Lab Values

Review

Pause
2:01 PM 04/08/2012

National Board of Medical Examiners - Google Chrom e

Q https://www.starttest.com/api/5.1.1.0/ITDStart.aspx?SVC=lbfd4732-460a-454b-90b5-99S5c0560007

Exam Section 1: Item 25 o f 50 H Mark

National Board o f Medical Examiners Comprehensive Basic Science Self-Assessment

Tim e Remaining: 3 hr 22 min 3 sec

extremities on the left, but they are more active in the upper extremity. Normal results of MR angiography are shown. This patient most likely has an occlusion of which of the following labeled arteries?

m A)

B)

C)

D)

E)

Previous

Next

Lab Values

Review

Pause
2:01 PM 04/08/2012

https://www.starttest.com/api/5.1.1.0/TTDStart.aspx?SVC=lbfd4732-460a-454b-90b5-9985c0560007

Exam Section 1: Item 26 o f 50 H Mark

National Board o f Medical Examiners Comprehensive Basic Science Self-Assessment

Tim e Remaining: 3 hr 20 min 43 sec

26. A 22-year-old woman comes to the physician because of numbness of her feet and poor balance for 3 months. She follows a vegan diet. She says, "I cant eat animal products, but I do try to keep my proteins and carbohydrates balanced." Physical examination shows absent ankle reflexes. Proprioception and sensation to vibration are decreased in both feet. Laboratory studies show a hemoglobin concentration of 11 g/dL and mean corpuscular volume of 110 pm 3 . This patient most likely has a dietary deficiency of which of the following? A) Folic acid B)lron C) Vitamin B6(pyridoxine) o D) Vitamin B 12(cobalamin) E) Vitamin C F) Vitamin E

N a tio n a l B o a rd o f M e d ic a l E xa m in e rs - G o o g le C h ro m e

i m

. _______________

['

cj

| 0

Q https://www.starttest.eom/api/5.l.1.0/TTDStart.aspx?SVC=lbfd4732-460a-454b-90b5-9985c0560007 Exam Section 1: Item 27 o f 50 H Mark National Board o f Medical Examiners Comprehensive Basic Science Self-Assessment Tim e Remaining: 3 hr 19 min 49 sec

27.

A 63-year-old man comes to the physician because of weakness and fatigue for 8 months. Physical examination shows massive splenomegaly. Laboratory studies show: Hemoglobin Hematocrit Leukocyte count Platelet count 8.2 g/dL 25% 6000/mm3 60,000/mm3

A peripheral blood smear shows the presence of myelocytes, metamyelocytes, nucleated erythrocytes, and teardrop erythrocytes. A bone marrow specimen shows markedly thickened trabeculae with replacement of the marrow by cellular fibrous tissue with admixed marrow elements. Which of the following is the most likely diagnosis? O A) Acute myelogenous leukemia B) Aplastic anemia C) Immune thrombocytopenic purpura D) Megaloblastic anemia o E) Myelofibrosis

LI

https://www.starttest.com/api/5.1.1.0/TTDStart.aspx?SVC=lbfd4732-460a-454b-90b5-9985c0560007

Exam Section 1: Item 28 o f 50 H Mark

National Board o f Medical Examiners Comprehensive Basic Science Self-Assessment

Tim e Remaining: 3 hr 17 min 40 sec

28. A 23-year-old primigravid woman at 18 weeks' gestation comes to the physician because she has felt minimal fetal movement during the past 2 weeks. There is no family history of congenital anomalies. Physical examination shows a uterus consistent in size with an 18-week gestation. Ultrasonography shows decreased amniotic fluid and normal-sized fetal kidneys, but the fetal bladder and ureters bilaterally appear markedly distended. The fetus appears to be male. Which of the following abnormalities best explains these findings? A) Penile hypospadias B) Placental insufficiency C) Polycystic kidney disease o D) Posterior urethral valves E) Prostatic nodular hyperplasia

https://www.starttest.com/api/5.1.1.0/TTDStart.aspx?SVC=lbfd4732-460a-454b-90b5-9985c0560007

Exam Section 1: Item 29 o f 50 H Mark

National Board o f Medical Examiners Comprehensive Basic Science Self-Assessment

Tim e Remaining: 3 hr 16 min 19 sec

29.

A 14-year-old boy collapses while running sprints during high school basketball tryouts and dies within several minutes. He was considered healthy and had no abnormal findings on prior physical examinations. His father died at the age of 38 years. At autopsy, the heart weighs 50% more than expected and has marked thickening of the left ventricular wall and the interventricular septum. There are no other gross abnormalities. Which of the following findings is also expected? A) Diffuse myocardial necrosis

o B) Disorganized hypertrophic cardiac myocytes C) Increased numbers of cardiac myocytes D) Interstitial infiltration of lymphocytes E) Premature coronary atherosclerosis

LI

https://www.starttest.com/api/5.1.1.0/TTDStart.aspx?SVC=lbfd4732-460a-454b-90b5-9985c0560007

Exam Section 1: Item 30 o f 50 H Mark

National Board o f Medical Examiners Comprehensive Basic Science Self-Assessment

Tim e Remaining: 3 hr 14 min 13 sec

30.

During an experiment, an investigator wishes to evaluate the serum cholesterol concentrations of all patients over the age of 50 years on the day of admission to the hospital. Both the mean and the median are the same. It is decided to use the mean as the measure of central tendency. Which of the following is the most appropriate measure of dispersion to analyze the data for this experiment?

O A) Coefficient of variation B) Interquartile range C) Percentile o D) Range E) Standard deviation

https:,7www.starttest.com/api/5.1.1.0/ITDStart.aspx?SVC=lbfd4732-460a-454b-90b5-9985c0560007

Exam Section 1: Item 31 o f 50 I Mark

National Board o f Medical Examiners Comprehensive Basic Science Self-Assessment

Tim e Remaining: 3 hr 13 min 4 sec

31. A 4-year-old boy is brought to the physician because of a 1-day history of restlessness, irritability, and intense perirectal itching. He attends a large day-care facility. Examination of the perirectal region shows small worms visualized under bright light. Cellophane tape test results for Enterobius vermicularis are positive. Which of the following is the most appropriate pharmacotherapy? C A) Azithromycin o B) Mebendazole C) Metronidazole C D) Pentamidine C E) Tetracycline

p 1
Previous Lab Values Review Pause ..ill 2:10 PM 04/08/2012

< >

LI

https://www.starttest.com/api/5.1.1.0/TTDStart.aspx?SVC=lbfd4732-460a-454b-90b5-9985c0560007

Exam Section 1: Item 32 o f 50 H Mark

National Board o f Medical Examiners Comprehensive Basic Science Self-Assessment

Tim e Remaining: 3 hr 10 min 12 sec

32.

A 49-year-old woman comes to the emergency department because of a 3-day history of fever, shortness of breath, and confusion. She is a postal worker. Her temperature is 38.4C (101.2F), respirations are 28/min, and blood pressure is 100/60 mm Hg. Physical examination shows nuchal rigidity. Breath sounds are decreased on the right side of the chest. A lumbar puncture is done. Analysis of cerebrospinal fluid (CSF) shows: Glucose Total protein Leukocyte count Segmented neutrophils Monocytes RBC 18mg/dL 138 mg/dL 638/mm3 87% 13% 2300/mm3

A Gram stain of CSF shows large, gram-positive, spore-forming bacilli. A chest x-ray shows marked widening of the mediastinum. Which of the following extracellular virulence factors most likely enables the causal organism to evade phagocytosis? A) Alginate

O B) Glucuronoxylomannan O C) Hyaluronic acid o D) Polyglutamic acid E) Polyribitol phosphate

LI

https://www.starttest.com/api/5.1.1.0/TTDStart.aspx?SVC=lbfd4732-460a-454b-90b5-9985c0560007

Exam Section 1: Item 33 o f 50 H Mark

National Board o f Medical Examiners Comprehensive Basic Science Self-Assessment

Tim e Remaining: 3 hr 9 min 57 sec

33.

A 40-year-old man with alcoholism is admitted to the hospital because of a 2-day history of confusion. Serum studies show a sodium concentration of 99 mEq/L. He is treated with 0.9% saline. Four days later, he develops slurred speech. Physical examination shows mild-to-moderate muscle weakness of all extremities and dysarthria. Sensation is intact. Babinski sign is present bilaterally. These findings are most likely caused by a lesion in which of the following locations? A) C) Bilateral cerebral hemispheres Medial diencephalon

o B) Brain stem O D) Muscle O E) Neuromuscular junction F) Peripheral nerve

LI

N a tio n a l B o a rd o f M e d ic a l E xa m in e rs - G o o g le C h ro m e

i m

. _______________

['

cj

| 0

Q https://www.starttest.eom/api/5.l.1.0/TTDStart.aspx?SVC=lbfd4732-460a-454b-90b5-9985c0560007 Exam Section 1: Item 34 o f 50 H Mark National Board o f Medical Examiners Comprehensive Basic Science Self-Assessment Tim e Remaining: 3 hr 5 min 41 sec

34. A 3-year-old boy is brought to the physician by his parents because of concerns about his ability to heal from injuries. They report that he develops bruises and scars with even mild trauma, particularly in comparison to their 5-year-old daughter. He was born at 35 weeks' gestation without complications. He has otherwise been healthy and has developed normally. Physical examination shows extensible skin, and the joints are freely mobile; there are thin, atrophic scars on both knees. A photograph of his hands is shown. Which of the following is the most likely cause of this patient's condition? A) Aberrant production of elastin B) Abnormal synthesis of extracellular glycoprotein o C) Defect in the synthesis of fibrillar collagen D) Dysfunctional cellular transport protein E) Normal age-related skin and joint development

https://www.starttest.com/api/5.1.1.0/TTDStart.aspx?SVC=lbfd4732-460a-454b-90b5-9985c0560007

Exam Section 1: Item 35 o f 50 H Mark

National Board o f Medical Examiners Comprehensive Basic Science Self-Assessment

Tim e Remaining: 3 hr 2 min 16 sec

35.

A randomized cohort study of drug X administered to subjects after a myocardial infarction found that overall there was no decrease in mortality compared with administration of a placebo after a myocardial infarction. However, on review of the data, there were statistically fewer deaths among drug X subjects in the subgroup with nontransmural myocardial infarction than in the placebo group. A retrospective assessment of the database available for drug X supported the observation. Which of the following is the most appropriate next step? A) Cross-sectional population study of administration of drug X vs. placebo after nontransmural myocardial infarction

o B) Prospective, randomized, controlled study of administration of drug X vs. placebo after nontransmural myocardial infarction C) Treatment of all patients with drug X after myocardial infarction D) Treatment of only patients with nontransmural myocardial infarction with drug X E) Treatment of only patients with transmural myocardial infarction with drug X

LI

N a tio n a l B o a rd o f M e d ic a l E xa m in e rs - G o o g le C h ro m e

i m

. _______________

['

cj

| 0

Q https://www.starttest.eom/api/5.l.1.0/TTDStart.aspx?SVC=lbfd4732-460a-454b-90b5-9985c0560007 Exam Section 1: Item 36 o f 50 H Mark National Board o f Medical Examiners Comprehensive Basic Science Self-Assessment Tim e Remaining: 3 hr 0 min 40 sec

36.

An 88-year-old man loses consciousness when his neck is palpated during a routine health maintenance examination. He has a 40-year history of hypertension that has been well controlled with hydrochlorothiazide and a 20-year history of degenerative osteoarthritis, for which he occasionally takes anti-inflammatory drugs. Which of the following is the most likely cause of the syncope?

O A) Complete heart block B) Laryngospasm C) Peripheral arteriole constriction D) Peripheral arteriole dilation o E) Sinus bradyarrhythmia F) Ventricular tachyarrhythmia

https://www.starttest.com/api/5.1.1.0/TTDStart.aspx?SVC=lbfd4732-460a-454b-90b5-9985c0560007

Exam Section 1: Item 37 o f 50 H Mark

National Board o f Medical Examiners Comprehensive Basic Science Self-Assessment

Tim e Remaining: 2 hr 55 min 23 sec

37.

An investigator is planning to create gene therapy for Leigh syndrome, which is caused by an A ^ G mutation in the mitochondrial tRNALeu gene. Which of the following is the most likely reason why mitochondria encode their own tRNA?

o A) Mitochondria cannot import proteins or RNA B) Mitochondria produce large amounts of reactive oxygen species C) Mitochondria use a non-standard genetic code D) The unusually high mitochondrial pH denatures nuclear-encoded tRNA E) The unusually low mitochondrial pH hydrolyzes nuclear-encoded tRNA

N a tio n a l B o a rd o f M e d ic a l E xa m in e rs - G o o g le C h ro m e

i m

. _______________

['

cj

| 0

Q https://www.starttest.eom/api/5.l.1.0/TTDStart.aspx?SVC=lbfd4732-460a-454b-90b5-9985c0560007 Exam Section 1: Item 38 o f 50 H Mark National Board o f Medical Examiners Comprehensive Basic Science Self-Assessment Tim e Remaining: 2 hr 55 min 19 sec

38. A 24-year-old man comes to the physician because of a 3-month history of inability to obtain an erection and occasional penile pain. He also has had mild chronic back pain for 2 years and hemorrhoids that bleed on occasion. He is married and has a 2-year-old son. He enjoys his job, and he and his wife have a good relationship. He lifts weights regularly and trains on his bicycle. He usually rides for 2 hours daily during the week and 5 to 6 hours daily on the weekend. On questioning, he does not recall having spontaneous nocturnal erections. Physical examination shows no abnormalities except for back tenderness and external hemorrhoids. Which of the following is the most likely cause of this patient's erectile dysfunction? O A) Compromised blood flow from the pampiniform plexus

e >

B) Damaged blood and nerve supply to the erectile tissue o C) Lumbosacral strain 0 D) Obturator nerve compression 0; E) Pressure of hemorrhoids on the erectile tissue

LI

? _____________________________
Q https: 7www.starttest.com/api/5.1.1.0/ITDStart.aspx?SVC=lbfd4732-460a-454b-90b5-9985c0560007 Exam Section 1: Item 39 of 50 I Mark National Board of Medical Examiners Comprehensive Basic Science Self-Assessment Time Remaining: 2 hr 53 min 47 sec

39. A 23-year-old woman at 32 weeks' gestation comes to the emergency department because of a 1-day history of left flank pain and fever. Her temperature is 39.1 C (102.3F), pulse is 104/min, respirations are 14/min, and blood pressure is 120/72 mm Hg. Physical examination shows prominent tenderness over the left costovertebral angle. A photomicrograph of a renal biopsy specimen from a similar patient is shown. Which of the following is the most likely diagnosis? o A) Acute pyelonephritis O B) Acute renal infarction C) Acute tubulointerstitial nephritis D) Crescentic glomerulonephritis O E) Hemolytic uremic syndrome

r*

s 1 r'

r ,

..

-A

Previous

Lab Values

Review

Pause
2:30 PM 04/08/2012

https://www.starttest.com/api/5.1.1.0/TTDStart.aspx?SVC=lbfd4732-460a-454b-90b5-9985c0560007

Exam Section 1: Item 40 o f 50 H Mark

National Board o f Medical Examiners Comprehensive Basic Science Self-Assessment

Tim e Remaining: 2 hr 51 min 32 sec

40.

A 17-year-old girl is brought to the physician because she has not had a menstrual period for 15 months. Menarche occurred at the age of 12 years. Menses had occurred at regular 28-day intervals until she began a weight-loss regimen 18 months ago. She is 165 cm (5 ft 5 in) tall and weighs 45 kg (100 lb); BMI is 17 kg/m2. She says that she eats only 800 calories daily and exercises for more than 3 hours daily. Which of the following serum findings is most likely in this patient? A) Decreased circadian variability of free thyroxine concentration

o B) Decreased gonadotropin-releasing hormone pulsatility C) Increased adrenocorticotropic hormone pulse frequency D) Increased diurnal variability of follicle-stimulating hormone concentration E) Reversal of diurnal variability of growth hormone concentration

LI

N a tio n a l B o a rd o f M e d ic a l E xa m in e rs - G o o g le C h ro m e

i m

. _______________

['

cj

| 0

https://ww w .starttest.eom /api/5.l.1.0/TTDStart.aspx?SVC=lbfd4732-460a-454b-90b5-9985c0560007

Exam Section 1: Item 41 o f 50 H Mark

National Board o f Medical Examiners Comprehensive Basic Science Self-Assessment

Tim e Remaining: 2 hr 51 min 29 sec

41. A 55-year-old woman with congestive heart failure has progressive shortness of breath and chest pain. An x-ray of the chest shows bilateral pleural effusions. Laboratory studies done on pleural fluid and serum show:

Pleural Fluid
Glucose Protein Lactate dehydrogenase Total nucleated cell count Which of the following is the most likely cause of the pleural effusion? A) Decreased lymphatic drainage B) Decreased oncotic pressure C) Decreased serum protein concentration o D) Increased hydrostatic pressure E) Increased vascular permeability 80 mg/dL 2 g/dL 25 U/L 500/mm3

Serum
100 mg/dL 7 g/dL 50 U/L

N a tio n a l B o a rd o f M e d ic a l E xa m in e rs - G o o g le C h ro m e

i m

. _______________

['

cj

| 0

https://ww w .starttest.eom /api/5.l.1.0/TTDStart.aspx?SVC=lbfd4732-460a-454b-90b5-9985c0560007

Exam Section 1: Item 42 o f 50 H Mark

National Board o f Medical Examiners Comprehensive Basic Science Self-Assessment

Tim e Remaining: 2 hr 49 min 49 sec

42.

One day after a total abdominal hysterectomy with salpingo-oophorectomy for stage III cervical cancer, a 42-year-old woman has abdominal distention. She has passed only 200 ml_ of urine since the operation. Ultrasonography shows an accumulation of fluid in the abdominal cavity; the physician suspects that the fluid is urine. This patient most likely sustained injury to the ureter during intraoperative ligation of which of the following arteries?

o A) Internal iliac artery B) Ovarian artery C) Pudendal artery C D) Superior vesicle arteries E) Ureteric branches of the renal artery

National Board of Medical Examiners - Google Chrom e

Q https://www.starttest.com/api/5.1.1.0/ITDStart.aspx?SVC=lbfd4732-460a-454b-90b5-9985c0560007 Exam Section 1: Item 43 o f 50 H Mark National Board o f Medical Examiners Comprehensive Basic Science Self-Assessment Tim e Remaining: 2 hr 48 min 53 sec

43. During an experiment using adult rats, bone formation and resorption are studied. The addition of compound X causes a decrease in bone resorption. Compound X is most likely which of the following? o A) Calcitonin O B) Parathyroid hormone C) Prednisone D) Thyroxine (T4) E) Tumor necrosis factor

Previous

Lab Values

Review

Help -

Pause

O
..ill

'< >

2:36 PM 04/08/2012

https://www.starttest.com/api/5.1.1.0/TTDStart.aspx?SVC=lbfd4732-460a-454b-90b5-9985c0560007

Exam Section 1: Item 44 o f 50 H Mark

National Board o f Medical Examiners Comprehensive Basic Science Self-Assessment

Tim e Remaining: 2 hr 46 min 43 sec

44.

An 18-year-old woman sustains second-degree burns over her right thigh and lower leg after spilling hot grease while working at a fast-food restaurant. Six weeks later, the surface area of the wound has markedly decreased. Which of the following processes best explains the wound contracture in this patient? A) Cross-linking of type II collagen fibers B) Inhibition of stromelysin-1 (matrix metalloproteinase-3)

o C) Myofibroblast activity O D) P-selectin activity E) Respiratory burst in macrophages

LI

https://www.starttest.com/api/5.1.1.0/TTDStart.aspx?SVC=lbfd4732-460a-454b-90b5-9985c0560007

Exam Section 1: Item 45 o f 50 H Mark

National Board o f Medical Examiners Comprehensive Basic Science Self-Assessment

Tim e Remaining: 2 hr 45 min 21 sec

45.

Osteogenesis imperfecta (01) is an autosomal dominant disorder characterized clinically by a remarkably wide variety of manifestations that includes fragile bones that fracture easily, blue sclerae, hearing loss, hyperextensible joints, and enamel dysplasia. Which of the following is the most likely explanation for the pleiotropic effects of the 01 gene?

O A) Contiguous gene deletion o B) Expression of a defective gene in multiple tissues O C) Loss of imprinting at multiple loci D) Somatic loss of heterozygosity in alleles E) Variable trinucleotide expansion

https://www.starttest.com/api/5.1.1.0/TTDStart.aspx?SVC=lbfd4732-460a-454b-90b5-9985c0560007

Exam Section 1: Item 46 o f 50 H Mark

National Board o f Medical Examiners Comprehensive Basic Science Self-Assessment

Tim e Remaining: 2 hr 44 min 25 sec

46. A healthy 28-year-old woman delivers a female newborn at term with a cranial malformation. There is no family history of similar findings. Physical examination of the newborn shows a large defect in the calvaria and meninges with a primitive brain. The newborn dies 2 days later. In this patient's future pregnancies, an increased maternal serum concentration of which of the following during the second trimester will most likely indicate the same malformation? o A) a-Fetoprotein O B) Free 3-human chorionic gonadotropin C) Inhibin A D) Pregnancy-associated plasma protein A O E) Unconjugated estriol

LI

N a tio n a l B o a rd o f M e d ic a l E xa m in e rs - G o o g le C h ro m e

i m

. _______________

['

cj

| 0

https://ww w .starttest.eom /api/5.l.1.0/TTDStart.aspx?SVC=lbfd4732-460a-454b-90b5-9985c0560007

Exam Section 1: Item 47 o f 50 H Mark

National Board o f Medical Examiners Comprehensive Basic Science Self-Assessment

Tim e Remaining: 2 hr 43 min 41 sec

47. A 2-month-old boy is brought to the physician for a follow-up examination. He was delivered at term to a 27-year-old woman with myasthenia gravis. Immediately after delivery, he developed respiratory distress that lasted for 2 weeks. Physical examination today shows no abnormalities. The most likely cause of this patient's transient disease is transplacental transfer of which of the following antibody types? A) IgA B)lgD O C) IgE < > D) IgG E)lgM

National Beard of Medical Examiners - Google Chrome Q https://www.starttest.com/api/5.1.1.0/ITDStart.aspx?SVC=lbfd4732-460a-454b-90b5-9985c0560007 Exam Section 1: Item 48 o f 50 | l^ ark National Board o f Medical Examiners Comprehensive Basic Science Self-Assessment Tim e Remaining: 2 hr 42 min 13 sec

48. A 1-year-old boy is brought to the physician for a well-child examination. Physical examination shows pallor. Laboratory studies show: Hemoglobin Hematocrit Erythrocyte count Mean corpuscular volume Reticulocyte count Leukocyte count Platelet count 8.5 g/dL 26% 2.1 million/mm3 65 |jm 3(N=70-86) 0.5% 8000/mm3with a normal differential 480,000/mm3

'O

g^

co

o
V : ^ 3

A peripheral blood smear is shown in the photomicrograph. Which of the following is the most likely diagnosis? A) Acute lymphoblastic leukemia B) Folic acid deficiency o C) Iron deficiency anemia D) Sickle cell disease E) Vitamin B 12 (cobalamin) deficiency

0 - n C % < L 0
J r \ 0

'

loda o&j
u o

N a tio n a l B o a rd o f M e d ic a l E xa m in e rs - G o o g le C h ro m e

i m

. _______________

['

cj

| 0

Q https://www.starttest.eom/api/5.l.1.0/TTDStart.aspx?SVC=lbfd4732-460a-454b-90b5-9985c0560007 Exam Section 1: Item 49 o f 50 H Mark National Board o f Medical Examiners Comprehensive Basic Science Self-Assessment Tim e Remaining: 2 hr 40 min 23 sec

49.

A 32-year-old woman comes to the physician because of intermittent abdominal cramps and diarrhea, alternating with constipation. The stools are loose and brown; there is no blood or mucus. She has had these symptoms for 15 years, but they have become more frequent during the past 3 months. She has not had fever or weight loss. Colonoscopy 3 years ago showed no abnormalities. She recently was promoted to a management position at her company. She is 160 cm (5 ft 3 in) tall and weighs 75 kg (165 lb); BMI is 29 kg/m2. Her temperature is 37C (98.6F), pulse is 72/min, respirations are 16/min, and blood pressure is 130/76 mm Hg. Examination shows no abnormalities. Test of the stool for occult blood is negative. Which of the following is the most likely diagnosis?

A) Colon cancer B) D) Inflammatory bowel disease Pancreatic insufficiency o C) Irritable bowel syndrome O E) Peptic ulcer disease

LI

National Board of Medical Examiners - Google Chrom e

Q h ttps://w w w .starttest.com /api/5.1 .1.0/TTDStart.aspx?SVC=lbfd4732-460a-454b-90b5-9985c0560007

Exam Section 1: Item 50 o f 50 H Mark

National Board o f Medical Examiners Comprehensive Basic Science Self-Assessment

Tim e Remaining: 2 hr 39 min 42 sec

50. A 45-year-old man with recently diagnosed early-onset Parkinson disease comes to the physician for a follow-up examination. He has a long history of major depressive disorder successfully treated with fluoxetine. Treatment with which of the following antiparkinsonian drugs is contraindicated in this patient? A) Amantadine C B) Benztropine C)Levodopa C D) Ropinirole o E) Selegiline

Previous

Lab Values

Review

Pause
2:45 PM 04/08/2012

N a tio n a l B o a rd o f M e d ic a l E xa m in e rs - G o o g le C h ro m e

i m

. _______________

['

cj

| 0

https://ww w .starttest.eom /api/5.l.1.0/TTDStart.aspx?SVC=lbfd4732-460a-454b-90b5-9985c0560007

Exam Section 2: Item 1 o f 50 H Mark

National Board o f Medical Examiners Comprehensive Basic Science Self-Assessment

Tim e Remaining: 4 hr 17 min 30 sec

1. A 35-year-old woman comes to the emergency department because of abdominal cramps, nausea, and vomiting for 8 hours. Her pulse is 106/min and regular, and blood pressure is 140/96 mm Hg. Physical examination shows rhinorrhea, excessive lacrimation, and diaphoresis. There is piloerection over most of the skin. The pupils are dilated, equal in size, and responsive to direct and indirect light. There is a resting tremor involving both upper extremities. During the examination, the patient is restless and yawns constantly. This patient is most likely experiencing withdrawal symptoms from which of the following substances? A) Cannabis

0 B) Cocaine O C) Diazepam O D) Ethanol o E) Heroin 0 F) Phenobarbital

https://www.starttest.com/api/5.1.1.0/TTDStart.aspx?SVC=lbfd4732-460a-454b-90b5-9985c0560007

Exam Section 2: Item 2 o f 50 H Mark

National Board o f Medical Examiners Comprehensive Basic Science Self-Assessment

Tim e Remaining: 4 hr 16 min 16 sec

2. A 58-year-old man from Eastern Europe who has recently immigrated to the USA with his family comes to the physician with his wife for an initial physical examination. Moderately severe emphysema is diagnosed, and continuous oxygen treatment is recommended. One week later, the wife calls the physician and tells him that her husband refuses to use the oxygen treatment. He is scheduled for a follow-up visit the next day. Which of the following is the most appropriate initial action for the physician to take during the next follow-up visit? A) Determine whether the patient is competent to make an informed decision B) Inform the patient about the deleterious effects of his decision o C) Initiate a discussion about the patient's concerns regarding the treatment D) Recognize that the couple is undergoing many adjustments and stresses and wait 6 months before discussing his noncompliance E) Recognize that the patient may be wary of the treatment due to his cultural background and enlist the aid of community members

LI

https://www.starttest.com/api/5.1.1.0/TTDStart.aspx?SVC=lbfd4732-460a-454b-90b5-9985c0560007

Exam Section 2: Item 3 o f 50 H Mark

National Board o f Medical Examiners Comprehensive Basic Science Self-Assessment

Tim e Remaining: 4 hr 15 min 24 sec

3. A 46-year-old man falls three stories from an open window and fractures the shaft of his right femur and pelvis. Thirty-six hours after stabilization of the fractures, he becomes confused and develops dyspnea, tachypnea, and arterial hypoxemia. Examination shows petechiae in the conjunctivae and over the pectoral regions. An x-ray of the chest shows a diffuse alveolar infiltrate. Pulmonary lavage shows cells staining for oil red 0 . Which of the following is the most likely cause of the respiratory dysfunction? A) Bacterial pneumonia B) Disseminated intravascular coagulation o C) Fat embolism D) Myocardial infarction O E) Pulmonary thromboembolism

LI

N a tio n a l B o a rd o f M e d ic a l E xa m in e rs - G o o g le C h ro m e

i m

. _______________

['

cj

| 0

Q https://www.starttest.eom/api/5.l.1.0/TTDStart.aspx?SVC=lbfd4732-460a-454b-90b5-9985c0560007 Exam Section 2: Item 4 o f 50 H Mark National Board o f Medical Examiners Comprehensive Basic Science Self-Assessment Tim e Remaining: 4 hr 13 min 28 sec

4. A 63-year-old man comes to the physician because of a 3-month history of abdominal pain that is only partially relieved by antacids. He states that he has also tried fasting, but it has had little to no effect on the pain. Palpation of the abdomen shows exquisite tenderness focused to the right of the midline. Laboratory studies show a fasting serum gastrin concentration of 800 pg/mL (N<100) and no evidence of Helicobacter pylori infection. Secretin infusion after the patient has fasted increases serum gastrin concentration to 1150 pg/mL. Secretion of which of the following is also most likely increased in this patient as the result of this infusion? O A) Ghrelin O B) Glucagon-like peptide-1 O C) Hepatic bile secretion o D) Pancreatic bicarbonate E) Saliva

N a tio n a l B o a rd o f M e d ic a l E xa m in e rs - G o o g le C h ro m e

i m

. _______________

['

cj

| 0

Q https://www.starttest.eom/api/5.l.1.0/TTDStart.aspx?SVC=lbfd4732-460a-454b-90b5-9985c0560007 Exam Section 2: Item 5 o f 50 H Mark National Board o f Medical Examiners Comprehensive Basic Science Self-Assessment Tim e Remaining: 4 hr 10 min 1 sec

5. A 25-year-old man with type 1 diabetes mellitus comes to the physician because of poor diabetic control for 3 months. He has had ketones in his urine, and his blood glucose concentrations have been in the 200-400 mg/dl_ range, particularly after exercise. He has been receiving two injections of insulin daily. Physical examination shows no abnormalities. His hemoglobin A 1c is 12%. Following exercise on a treadmill for 45 minutes, the patient has an increase in his serum glucose concentration from 175 m g/dLto 225 mg/dl_. Which of the following hepatic enzymes is most likely activated initially by exercise in this patient? O A) Debranching enzyme O B) Glycogen synthase C) Phospholipase C o D) Phosphorylase kinase O E) Protein kinase B F) Protein kinase G G) Receptor protein-tyrosine kinase

N a tio n a l B o a rd o f M e d ic a l E xa m in e rs - G o o g le C h ro m e

i m

. _______________

['

cj

| 0

Q https://www.starttest.eom/api/5.l.1.0/TTDStart.aspx?SVC=lbfd4732-460a-454b-90b5-9985c0560007 Exam Section 2: Item 6 o f 50 H Mark National Board o f Medical Examiners Comprehensive Basic Science Self-Assessment Tim e Remaining: 4 hr 7 min 34 sec

6.

A 1-year-old boy is brought to the physician by his mother because of failure to thrive. He has had intermittent respiratory tract infections and diarrhea during the past 6 months. He was born at term following an uncomplicated pregnancy. He has received only diluted cow's milk and a homemade gruel of rice, sugar, and water since the age of 3 months. His vital signs are normal. He is at the 30th percentile for weight. He appears apathetic. Physical examination shows dry skin, moderate hepatomegaly, a distended abdomen, an umbilical hernia protruding 2 cm, and 2+ pitting edema of the lower back and face. There is no jaundice. His hair is easily plucked. Laboratory studies show pancytopenia. Which of the following is the most likely cause of his symptoms?

O A) Cystic fibrosis B) Enterobacterial infection C) Extrahepatic biliary atresia D) Hepatolenticular degeneration (Wilson disease) o E) Protein-calorie malnutrition

Jp National Board of Medical Examiners - Google Chrom e g https: 7 www.starttest.corn/api/5.1.1.0/ITDStart.aspx?SVC=lbfd4732-460a-454b-90b5-9985c0560007

Exam Section 2: Item 7 o f 50 0 Mark

National Board o f Medical Examiners Comprehensive Basic Science Self-Assessment

Tim e Remaining: 4 hr 0 min 29 sec

7. A 66-year-old man with type 2 diabetes mellitus and hypertension is brought to the emergency department 30 minutes after the sudden onset of left eyelid drooping, double vision, and mild weakness of the right hand and leg. His pulse is 88/min and regular, and blood pressure is 159/99 mm Hg. Examination of the head shows a substantially droopy left eyelid, and in primary gaze, the left eye is exotropic and somewhat lower than the right. He has slowed finger movements on the right, a pronator drift with the right hand, and mild hyperreflexia on the right. Which of the following is the most likely site and diagnosis of the patient's lesion? A) Angular gyrus (Gerstmann syndrome) B) Dorsolateral thalamus (Dejerine-Roussy syndrome) C) Lower medulla (Wallenberg syndrome) o D) Lower midbrain (Weber syndrome) O E) Upper spinal cord (Horner syndrome)

Previous

Lab Values

Review

Help

I*
-

Pause

O
..in

Tb

r ~ r :

< >

3:11 PM 04/08/2012

National Board of Medical Examiners - Google Chrom e Q https://wvw.starttest.com /api/511M TDStart.aspx7SVC3lbfd4732-460a-454b-90b5-9985c0560007

I___

Exam Section 2: Item 8 o f 50 I Mark

National Board o f Medical Examiners Comprehensive Basic Science Self-Assessment

Tim e Remaining: 3 hr 59 min 35 sec

8. A study is conducted to assess the relationship between age and serum total cholesterol concentration in three groups. Group X consists of 50 children (ages 6 to 10 years), Group Y consists of 50 adolescents (ages 12 to 16 years), and Group Z consists of 50 young adults (ages 21 to 25 years). The fasting serum total cholesterol concentration is measured in all participants. Which of the following statistical tests is most appropriate to compare the serum total cholesterol concentration results among Groups X, Y, and Z in this study? = > A) Analysis of variance o B) Chi-square test 0 C) Independent West D) Multiple logistic regression E) Paired West C F) Pearson correlation

[^f National Board of Medical Examiners - Google Chrom e

Q https://www.starttest.com/api/5.1.1.0/TTDStart.aspx?SVC=lbfd4732-460a-454b-90b5-9985c0560007 Exam Section 2: Item 9 o f 50 H Mark National Board o f Medical Examiners Comprehensive Basic Science Self-Assessment Tim e Remaining: 3 hr 56 min 41 sec

9. The graph shows the response elicited by different concentrations of drug X in a system containing spare receptors in the absence (solid curve) and presence (dashed curves) of two different concentrations of drug Y. Drug Y alone has no effect. Which of the following best describes drug Y? A) Competitive reversible antagonist

O B) Full agonist O C) Inverse agonist = > D) Noncompetitive antagonist o E) Partial agonist

[Drug X] (pM) [Drug Y] = 0 uM [Drug Y] = 1 uM [Drug Y] = 10 pM

Previous

Lab Values

Review

Pause
3:15 PM 04/08/2012

[^f National Board of Medical Examiners - Google Chrom e Q https://www.starttest.com/api/5.1.1.0/TTDStart.aspx?SVC=lbfd4732-460a-454b-90b5-9985c0560007

Exam Section 2: Item 10 o f 50 I Mark

National Board o f Medical Examiners Comprehensive Basic Science Self-Assessment

Tim e Remaining: 3 hr 54 min 47 sec

10. A 72-year-old woman has a persistent headache, weakness of the lower extremities, and increased intracranial pressure. Cerebral arteriography shows a neoplasm causing inferior displacement of the artery at the location indicated by the arrows shown. Which of the following is the most likely displaced artery? o A) Anterior cerebral B) Basilar O C) Middle cerebral O D) Posterior cerebral O E) Vertebral

p 1
Lab Values Review
3:17 PM 04/08/2012

National Board of Medical Examiners - Google Chrom e Q https://www.starttest.com/api/5.1.1.0/ITDStart.aspx?SVC=lbfd4732-460a-454b-90b5-9985c0560007

I___

Exam Section 2: Item 11 o f 50 0 Mark

National Board o f Medical Examiners Comprehensive Basic Science Self-Assessment

Tim e Remaining: 3 hr 52 min 51 sec

11. A 24-year-old man is brought to the physician by his wife because of a 2-day history of progressive confusion. His temperature is 38.3C (101 F). Physical examination shows no other abnormalities. When he speaks, he enunciates clearly, and his phrasing is of average length. However, he uses words and word-like utterances in a manner that makes little sense. He does not follow any commands. Which of the following is the most likely diagnosis? A) Cerebral toxoplasmosis B) Herpes simplex encephalitis O C) HIV encephalopathy o D) Meningococcal meningitis O E) Subdural empyema

[^p National Board of Medical Examiners - Google Chrom e Q https://www.starttest.com/api/5.1.1.0/TTDStart.aspx?SVC=lbfd4732-460a-454b-90b5-9985c0560007

I'

Exam Section 2: Item 12 o f 50 I Mark

National Board o f Medical Examiners Comprehensive Basic Science Self-Assessment

Tim e Remaining: 3 hr 50 min 26 sec

12.

During an experiment with laser scanning microscopy, it is observed that bone marrow-derived stem cells migrate through the thymus. Which of the following is most likely to occur as a result of this process?

O A) CD40 induction O B) Germinal center formation C) Membrane immunoglobulin expression D) Thymic involution o E) T-lymphocyte antigen receptor selection

Previous

Lab Values

Review

Help -

Pause

O
.,il, $ < >
3:22 PM 04/08/2012

National Board of Medical Examiners - Google Chrom e Q https://wvw.starttest.com /api/511M TDStart.aspx7SVC3lbfd4732-460a-454b-90b5-9985c0560007

I___

Exam Section 2: Item 13 o f 50 I Mark

National Board o f Medical Examiners Comprehensive Basic Science Self-Assessment

Tim e Remaining: 3 hr 49 min 21 sec

13.

A 43-year-old woman with systemic lupus erythematosus has nephrotic syndrome. Examination of renal tissue obtained on biopsy shows a diffuse proliferative glomerulonephritis with electron-dense deposits along the glomerular basement membrane. Immunofluorescent studies show granular deposits of complement along the basement membrane. Which of the following is the most likely cause of these findings? A) Antibodies directed against glomerular basement membrane B) Antibodies directed against viral antigens expressed on endothelial cells

o C) Anti-DNA/DNA immune complex deposition in glomeruli D) Autoantibodies to podocyte antigens E) CD8+ T-lymphocyte cytotoxicity of virus-infected mesangial cells

[^p National Board of Medical Examiners - Google Chrom e Q https://ww w .starttest.eom /api/5.l.1.0/TTDStart.aspx?SVC=lbfd4732-460a-454b-90b5-9985c0560007

Exam Section 2: Item 14 o f 50 I Mark

National Board o f Medical Examiners Comprehensive Basic Science Self-Assessment

Tim e Remaining: 3 hr 48 min 28 sec

14. A 52-year-old man with a family history of cardiovascular disease begins treatment with 81-mg aspirin for cardioprotection. This protective effect is best explained by which of the following actions of this drug on platelets? A) Irreversible inhibition of prostacyclin (PGI2) production B) Irreversible inhibition of prostaglandin E2(PGE2) production o C) Irreversible inhibition of thromboxane A 2 (TXA2) production O D) Reversible inhibition of PGE2 production O E) Reversible inhibition of PGI2 production 0 F) Reversible inhibition of TXA2 production

Previous

Lab Values

Review

Help -

Pause

O
..in

< >

3:24 PM 04/08/2012

National Board of Medical Examiners - Google Chrom e Q https://www.starttest.com/api/5.1.1.0/ITDStart.aspx?SVC=lbfd4732-460a-454b-90b5-9985c0560007

Exam Section 2: Item 15 o f 50 I Mark

National Board o f Medical Examiners Comprehensive Basic Science Self-Assessment

Tim e Remaining: 3 hr 47 min 33 sec

15. A 50-year-old man is brought to the emergency department 45 minutes after he was found unresponsive at home. He has a 3-day history of progressive lethargy and increasingly severe headache. He hit his head in a fall 4 weeks ago; at that time, he did not seek medical attention. He appears stuporous. His pulse is 60/min, and blood pressure is 150/90 mm Hg. Neurologic examination shows right hemiparesis. A CT scan of the head without contrast is shown. The most likely cause of this patient's condition is bleeding from which of the following structures? A) Arteriovenous malformation

C B) Berry aneurysm o C) Bridging cortical vein D) Middle meningeal artery E) Superior sagittal sinus

[^p National Board of Medical Examiners - Google Chrom e Q https://www.starttest.com/api/5.1.1.0/TTDStart.aspx?SVC=lbfd4732-460a-454b-90b5-9985c0560007

I'

Exam Section 2: Item 16 o f 50 0 Mark

National Board o f Medical Examiners Comprehensive Basic Science Self-Assessment

Tim e Remaining: 3 hr 46 min 30 sec

16. A 63-year-old woman with an 18-year history of type 2 diabetes mellitus has decreased visual acuity. Funduscopic examination of both eyes shows hemorrhages and "cotton wool" spots. Which of the following is the predominant underlying pathologic condition in this disease? O A) Atherosclerosis o B) Microangiopathy O C) Optic neuritis O D) Thrombosis O E) Vasculitis

Previous

Lab Values

Review

Help
-

Pause

O
.,il,

3:26 PM
< >

04/08/2012

National Board of Medical Examiners - Google Chrom e Q https://www.starttest.com/api/5.1.1.0/ITDStart.aspx?SVC=lbfd4732-460a-454b-90b5-9985c0560007

I___

Exam Section 2: Item 17 o f 50 I Mark

National Board o f Medical Examiners Comprehensive Basic Science Self-Assessment

Tim e Remaining: 3 hr 45 min 32 sec

17. A previously healthy 29-year-old man is brought to theemergency department 1hour after colliding with a barrier while snowboarding.Physicalexaminationshows edema and deformity of the right upper extremity. An x-ray of the affectedextremity shows adisplaced fracture of the surgical neck of the humerus. Thispatient is most likely to have impaired or absent sensation in which of the following locations ipsilateral to the fracture? o A) Lateral aspect of the arm B) Medial aspect of the arm C) Anterior aspect of the mid forearm D) Posterior aspect of the mid forearm 0 E) Pad of the thumb 0 F) Pad of the ring finger

[^p National Board of Medical Examiners - Google Chrom e Q https://www.starttest.com/api/5.1.1.0/TTDStart.aspx?SVC=lbfd4732-460a-454b-90b5-9985c0560007

I'

Exam Section 2: Item 18 o f 50 I Mark

National Board o f Medical Examiners Comprehensive Basic Science Self-Assessment

Tim e Remaining: 3 hr 42 min 45 sec

18. An investigator is developing a new drug, Drug X, to protect health care workers after accidental inoculation with blood containing HIV. The drug is designed to block viral entry into the cell. Which of the following is the most appropriate target for Drug X? A) CD8 o B) Chemokine receptor 0 C) Fc receptor

0 D) HIV protease 0 E)lntegrase 0 F) lnterleukin-2 (IL-2) receptor G) Reverse transcriptase 0 H) Tat protein

Previous

Lab Values

Review

Help -

Pause

O
..in

< >

3:30 PM 04/08/2012

[^f National Board of Medical Examiners - Google Chrom e

-I

https://www.starttest.com/api/5.1.1.0/ITDStart.aspx?SVC=lbfd4732-460a-454b-90b5-9985c0560007

Exam Section 2: Item 19 o f 50 H Mark

National Board o f Medical Examiners Comprehensive Basic Science Self-Assessment

Tim e Remaining: 3 hr 40 min 48 sec

19. A 59-year-old man comes to the physician because of a 3-day history of yellow-tinted eyes and intermittent regurgitation of small amounts of blood. He has had progressive fatigue and increased abdominal girth during the past month. Physical examination shows scleral icterus, pale conjunctivae, and a protuberant abdomen. An abdominal fluid wave is palpated. Laboratory studies show: Hemoglobin Hematocrit Mean corpuscular hemoglobin concentration Mean corpuscular volume Serum Alkaline phosphatase AST ALT 9.8 g/dL 29% 28% Hb/cell 70 pm3 210 U/L 20 U/L 10 U/L

..... r . ^ *

v r.' V^ y-> ' A * ],,.

. :

*'% - /?., *
>

4:

'\8rMsA E j** :** K .:


*7.

s'

i.

i * 4

i
t*

/* jc'-r
k * 4

a
'S
< *

I ^ p *^

A biopsy specimen of tissue from the liver is shown in the photomicrograph. Upper gastrointestinal endoscopy is most likely to show which of the following? o A) Esophageal varices B) Hemorrhagic gastritis C) Hiatal hernia D) Mallory-Weiss laceration O E) Schatzki ring

V .f
m m tm A '.E

Previous

p 1
Lab Values

Review -

Pause

C l
.,il, ^ < > 3:32 PM 04/08/2012

LI

National Board of Medical Examiners - Google Chrom e Q https://www.starttest.com/api/5.1.1.0/ITDStart.aspx?SVC=lbfd4732-460a-454b-90b5-9985c0560007

I___

Exam Section 2: Item 20 o f 50 0 Mark

National Board o f Medical Examiners Comprehensive Basic Science Self-Assessment

Tim e Remaining: 3 hr 36 min 5 sec

20. A 28-year-old woman wants to lose weight. She is 160 cm (5 ft 3 in) tall and weighs 81.5 kg (180 lb); BMI is 32 kg/m2. She consumes 1800 calories daily and has a sedentary life-style. Assume that there are 3500 calories per pound and that brisk walking consumes 500 calories per hour. Which of the following regimens is most likely to help her lose 0.9 kg (2 lb) weekly?

A) B) C) D) E) F)

Calories Per Day Decrease by 100 Decrease by 100 Decrease by 500 Decrease by 500
No change No change

Brisk Walking Daily


1 /4 hour 1 hour 1 /4 hour 1 hour 1 /4 hour 1 hour

National Board of Medical Examiners - Google Chrom e Q https://www.starttest.com/api/5.1.1.0/ITDStart.aspx?SVC=lbfd4732-460a-454b-90b5-9985c0560007

I___

Exam Section 2: Item 21 o f 50 I Mark

National Board o f Medical Examiners Comprehensive Basic Science Self-Assessment

Tim e Remaining: 3 hr 34 min 43 sec

21. A 20-year-old man who is an army recruit comes to the physician because of an 8-hour history of headache, stiff neck, and rash. His temperature is 40C (104F), pulse is 120/min, respirations are 25/min, and blood pressure is 90/50 mm Hg. Physical examination shows nuchal rigidity and a petechial rash over the extremities. A lumbar puncture is done. Culture of cerebrospinal fluid grows gram-negative diplococci. Which of the following would be used to further classify this causal organism into a serogroup? o A) Antibodies to capsular polysaccharide B) Comparison of aerobic and anaerobic growth C) Hemolytic pattern on sheep blood agar D) Measurement of total genome size O E) Phage typing

National Board of Medical Examiners - Google Chrom e Q https://www.starttest.com/api/5.1.1.0/ITDStart.aspx?SVC=lbfd4732-460a-454b-90b5-9985c0560007

Exam Section 2: Item 22 o f 50 I Mark

National Board o f Medical Examiners Comprehensive Basic Science Self-Assessment

Tim e Remaining: 3 hr 32 min 56 sec

22. A 17-year-old boy is brought to the emergency department because of a high-grade fever and increasingly severe diffuse abdominal pain during the past 3 hours. The pain started 2 days ago, but it resolved without medical treatment until hours prior to admission, when the abdominal pain recurred. He appears anxious, pale, and diaphoretic. His temperature is 38C (100.4F). Abdominal examination shows guarding and rebound tenderness. An exploratory operation is done. A sample of fluid obtained from the abdominal cavity is found to have a specific gravity greater than 1.020, numerous leukocytes (mainly segmented neutrophils), and cellular debris. Which of the following best describes this fluid? A) Ascites B) Blood C) Exudate D) Lymphedema o E) Transudate

National Board of Medical Examiners - Google Chrom e Q https://www.starttest.com/api/5.1.1.0/ITDStart.aspx?SVC=lbfd4732-460a-454b-90b5-9985c0560007

I___

Exam Section 2: Item 23 o f 50 I Mark

National Board o f Medical Examiners Comprehensive Basic Science Self-Assessment

Tim e Remaining: 3 hr 31 min 17 sec

23. A 41-year-old man comes to the physician's office because of recurrent headaches, flushing, and palpitations for the past 3 months. Blood pressure measurements have ranged from 130/76 mm Hg to 200/120 mm Hg. CT scan of the abdomen shows a right suprarenal mass. An increase in the serum concentration of which of the following hormones is the most likely cause of the episodic hypertension in this patient? O A) Aldosterone

0 B) Angiotensin II 0 C) Cortisol 0 D) Endothelin


o E) Epinephrine

https://www.starttest.com/api/5.1.1.0/TTDStart.aspx?SVC=lbfd4732-460a-454b-90b5-9985c0560007

Exam Section 2: Item 24 o f 50 H Mark

National Board o f Medical Examiners Comprehensive Basic Science Self-Assessment

Tim e Remaining: 3 hr 29 min 11 sec

24. Two programs for the treatment of patients with newly detected hypercholesterolemia were tried in a community. Program A was used in one district of the community, and Program B was used in another. After four years, 45% of the 2200 patients on Program A and 49% of the 1900 patients on Program B had been successfully treated for hypercholesterolemia. The difference between the success rates for the two programs was statistically significant (p < .01). Health officials, however, decided not to change to Program B in the first district because the magnitude of the difference was so small. Which of the following best explains their decision? o A) They attributed the difference in success rates to chance alone B {> B) They distinguished between statistical significance and practical importance of the difference in success rates D) They felt the samples were too small to justify a decision in favor of Program B C) They felt the p value was too small to justify a decision in favor of Program B

LI

National Board of Medical Examiners - Google Chrom e Q https://www.starttest.com/api/5.1.1.0/ITDStart.aspx?SVC=lbfd4732-460a-454b-90b5-9985c0560007

I___

Exam Section 2: Item 25 o f 50 I Mark

National Board o f Medical Examiners Comprehensive Basic Science Self-Assessment

Tim e Remaining: 3 hr 28 min 26 sec

25.

A 22-year-old woman comes to the physician because of a 5-day history of white spots over her upper torso. Two days ago, she returned from a vacation at the beach, where she noticed that certain areas of her chest and back did not tan. Her vital signs are within normal limits. Physical examination shows 2-cm, hypopigmented macules with areas of fine scaling over the chest, back, and proximal upper extremities. Microscopic examination of a scraping from one of the macules shows hyphae and yeast forms. Which of the following is the most likely causal organism? A) Candida albicans B) Cladosporium cladosporioides

o C) Maiassezia furfur
G D) Microsporum canis E) Sporothrix schenckii

National Board of Medical Examiners - Google Chrom e Q https://www.starttest.com/api/5.1.1.0/ITDStart.aspx?SVC=lbfd4732-460a-454b-90b5-9985c0560007

I___

Exam Section 2: Item 26 o f 50 0 Mark

National Board o f Medical Examiners Comprehensive Basic Science Self-Assessment

Tim e Remaining: 3 hr 26 min 46 sec

26. A 50-year-old man comes to the physician for a follow-up examination. He has been receiving nortriptyline twice daily for major depressive disorder. The dosage of his medication was doubled from once to twice daily 2 months ago because the initial dosage was ineffective. At this visit, the patient tells the physician that his symptoms have not improved. Mental status examination shows a flat affect. Laboratory studies show a plasma nortriptyline concentration 30% of the lower end of the normal range. Nortriptyline is eliminated mainly by hepatic metabolism catalyzed by the cytochrome P450 2D6 (CYP2D6). Which of the following best explains the patient's lack of clinical response? O A) Induction of CYP2D6 by nortriptyline o B) Inheritance of an amplified CYP2D6 locus C) Inheritance of two inactive CYP2D6 alleles O D) Inhibition of CYP2D6 by nortriptyline E) Pharmacodynamic tolerance

[^p National Board of Medical Examiners - Google Chrom e Q https://www.starttest.com/api/5.1.1.0/TTDStart.aspx?SVC=lbfd4732-460a-454b-90b5-9985c0560007

I'

Exam Section 2: Item 27 o f 50 I Mark

National Board o f Medical Examiners Comprehensive Basic Science Self-Assessment

Tim e Remaining: 3 hr 25 min 4 sec

27. A 41 -year-old woman is brought to the emergency department by her husband because of pelvic pain for 6 hours. She also has had heavy menstrual periods during the past 3 months. A CT scan of the abdomen shows a tumor in the left ovary. An oophorectomy is scheduled. During the procedure, which of the following structures immediately deep to the infundibulopelvic (suspensory) ligament must be protected while isolating and ligating the ovarian vessels in this ligament? O A) Hypogastric nerve 0 B) Lumbosacral trunk 0 C) Round ligament D) Sympathetic trunk o E) Ureter

Previous

Lab Values

Review

Help
-

Pause

O
.,il,

3:48 PM
< >

04/08/2012

National Board of Medical Examiners - Google Chrom e Q https://www.starttest.com/api/5.1.1.0/ITDStart.aspx?SVC=lbfd4732-460a-454b-90b5-9985c0560007

I___

Exam Section 2: Item 29 o f 50 I Mark

National Board o f Medical Examiners Comprehensive Basic Science Self-Assessment

Tim e Remaining: 3 hr 21 min 32 sec

29. A female newborn is delivered at 34 weeks' gestation. Her temperature is 37.4C (99.3F), pulse is 96/min, respirations are 20/min, and blood pressure is 108/72 mm Hg. Physical examination shows microcephaly and bilateral cataracts; there is no cyanosis. There is a continuous "machinery-like" murmur increasing in intensity during systole and waning during diastole; it is loudest in the second left intercostal space and radiates down the left sternal border. The apical impulse is prominent, and there is a thrill in the second left intercostal space. The physician suspects that the findings in this patient were caused by a congenital infection. Which of the following infectious agents is the most likely cause? O A) Cytomegalovirus O B) Epstein-Barr virus O C) Herpes simplex virus D) HIV

Mycobacterium tuberculosis o F) Rubella virus


E) O G) Toxoplasma gondii H) Treponema pallidum

National Board of Medical Examiners - Google Chrom e Q https://www.starttest.com/api/5.1.1.0/ITDStart.aspx?SVC=lbfd4732-460a-454b-90b5-9985c0560007

Exam Section 2: Item 29 o f 50 I Mark

National Board o f Medical Examiners Comprehensive Basic Science Self-Assessment

Tim e Remaining: 3 hr 20 min 16 sec

29. A female newborn is delivered at 34 weeks' gestation. Her temperature is 37.4C (99.3F), pulse is 96/min, respirations are 20/min, and blood pressure is 108/72 mm Hg. Physical examination shows microcephaly and bilateral cataracts; there is no cyanosis. There is a continuous "machinery-like" murmur increasing in intensity during systole and waning during diastole; it is loudest in the second left intercostal space and radiates down the left sternal border. The apical impulse is prominent, and there is a thrill in the second left intercostal space. The physician suspects that the findings in this patient were caused by a congenital infection. Which of the following infectious agents is the most likely cause? O A) Cytomegalovirus O B) Epstein-Barr virus O C) Herpes simplex virus D) HIV

Mycobacterium tuberculosis o F) Rubella virus


E) O G) Toxoplasma gondii H) Treponema pallidum

National Board of Medical Examiners - Google Chrom e Q https://www.starttest.com/api/5.1.1.0/ITDStart.aspx?SVC=lbfd4732-460a-454b-90b5-9985c0560007

Exam Section 2: Item 30 o f 50 0 Mark

National Board o f Medical Examiners Comprehensive Basic Science Self-Assessment

Tim e Remaining: 3 hr 18 min 13 sec

30.

A 5-year-old girl is brought to the physician because of a 1-year history of generalized tonic-clonic seizures that occur more frequently when she has a fever. Her 3-year-old brother has had similar episodes. Neurologic examination shows no abnormalities. Genetic testing shows a mutation affecting one of the subunits of the Y-aminobutyric acidA receptor. Which of the following mechanisms is the most likely cause of seizures in this patient? A) Decreased postsynaptic calcium influx

o B) Decreased postsynaptic chloride influx C) Decreased presynaptic calcium influx D) Increased postsynaptic potassium influx E) Increased presynaptic chloride influx F) Increased presynaptic potassium influx

N a tio n a l B e a rd o f M e d ic a l E xa m in e rs - G o o g le C h ro m e

https://ww w .starttest.eom /api/5.l.1.0/TTDStart.aspx?SVC=lbfd4732-460a-454b-90b5-9985c0560007

Exam Section 2: Item 31 o f 50 H Mark

National Board o f Medical Examiners Comprehensive Basic Science Self-Assessment

(g/ease Wait

31.

A 63-year-old man with alcoholism is diagnosed with hepatic encephalopathy. Treatment with lactulose therapy via nasogastric tube is begun. The effectiveness of this intervention requires which of the following mechanisms? A) Absorbed lactulose binding alcohol in the blood, decreasing blood alcohol concentrations B) Lactulose binding of ingested alcohol within the colonic lumen for excretion C) Metabolism to glucose and galactose, increasing serum glucose concentrations D) Osmotic diarrhea, flushing out ingested alcohol from the gut

o E) Trapping of ammonia in the colon by acidic metabolites of lactulose

National Board of Medical Examiners - Google Chrom e Q https://www.starttest.com/api/5.1.1.0/ITDStart.aspx?SVC=lbfd4732-460a-454b-90b5-9985c0560007

I___

Exam Section 2: Item 32 o f 50 I Mark

National Board o f Medical Examiners Comprehensive Basic Science Self-Assessment

Tim e Remaining: 3 hr 15 min 59 sec

32.

During a 5-year study at a hospital, data about antimicrobial drug use for respiratory infections and antimicrobial susceptibility for respiratory isolates are compiled. At the end of 5 years, the data show that levofloxacin was the most common drug used to treat respiratory infections and that its usage increased significantly since the beginning of the study. Resistance of Klebsiella pneumoniae to levofloxacin increased from 20% to 54% during the 5-year period. Based on these data, there is a greater likelihood of detecting which of the following in the latter isolates compared with the bacteria recovered at the beginning of the study? A) Expression of AmpC 3-lactamases B) Expression of carbapenemase C) Expression of extended-spectrum (3-lactamases D) Mutation of the gene encoding dihydrofolate reductase E) Mutation of the gene encoding elongation factor-2

o F) Mutation of the gene encoding topoisomerase II

National Board of Medical Examiners - Google Chrom e Q https://www.starttest.com/api/5.1.1.0/ITDStart.aspx?SVC=lbfd4732-460a-454b-90b5-9985c0560007

I___

Exam Section 2: Item 33 o f 50 I Mark

National Board o f Medical Examiners Comprehensive Basic Science Self-Assessment

Tim e Remaining: 3 hr 15 min 21 sec

33. A 32-year-old man sustains a head injury in a motorcycle accident and is admitted to the intensive care unit. Six days later he becomes confused and is very thirsty. His serum sodium concentration is 158 mEq/L, and his urine output is 6.1 L/day. These findings most likely indicate that he has a disorder that is affecting which of the following labeled parts of the nephron shown?

National Board of Medical Examiners - Google Chrom e Q https://wvw.starttest.com /api/511M TDStart.aspx7SVC3lbfd4732-460a-454b-90b5-9985c0560007

I___

Exam Section 2: Item 34 o f 50 I Mark

National Board o f Medical Examiners Comprehensive Basic Science Self-Assessment

Tim e Remaining: 3 hr 14 min 32 sec

34. A 70-year-old man is admitted to the hospital because of painless jaundice for the past 2 weeks. He has had dark urine and white stools for the past 7 days. Examination of the abdomen discloses no abnormalities. A CT scan of the abdomen shows a large poorly defined soft tissue density in the head of the pancreas. Which of the following is the most likely cause of the jaundice? A) Cholelithiasis o B) Common bile duct obstruction O C) Liver metastases O D) Pancreatic duct obstruction E) Porta hepatis metastases

National Board of Medical Examiners - Google Chrom e Q https://www.starttest.com/api/5.1.1.0/ITDStart.aspx?SVC=lbfd4732-460a-454b-90b5-9985c0560007

I___

Exam Section 2: Item 35 o f 50 0 Mark

National Board o f Medical Examiners Comprehensive Basic Science Self-Assessment

Tim e Remaining: 3 hr 12 min 48 sec

35. A 75-year-old man comes to the physician because of a 1-month history of leg swelling. His respirations are 20/min. Physical examination shows jugular venous distention. There is 3+ pitting edema of the lower extremities below the knees. The lungs are clear to auscultation. Echocardiography is most likely to show an enlargement of which of the following vascular structures? A) C) Inferior vena cava and left atrium Right atrium and left atrium

O B) Left atrium and left ventricle o D) Right atrium and right ventricle O E) Right ventricle and left atrium

National Board of Medical Examiners - Google Chrom e Q https://www.starttest.com/api/5.1.1.0/ITDStart.aspx?SVC=lbfd4732-460a-454b-90b5-9985c0560007

Exam Section 2: Item 36 o f 50 0 Mark

National Board o f Medical Examiners Comprehensive Basic Science Self-Assessment

Tim e Remaining: 3 hr 10 min 54 sec

36.

A 10-year-old boyis brought to the physicianby his parents for a follow-up examination because of mental retardation and pigmentary anomalies. Heunderwent an operation at the age of 3 years to correctsyndactyly between the middle and ring fingers bilaterally. Physical examinationshows streaky hyperpigmentation. Chromosomal analysis shows 46,XY in 15 cells and 69,XXY in 5 cells. Which of the following is the most likely explanation for the karyotype findings in this patient?

C A) Deletion o B) Duplication C) Inversion D) Mosaicism E) Ring chromosome F) Translocation

[^p National Board of Medical Examiners - Google Chrom e Q https://ww w .starttest.eom /api/5.l.1.0/TTDStart.aspx?SVC=lbfd4732-460a-454b-90b5-9985c0560007

I'

Exam Section 2: Item 37 o f 50 I Mark

National Board o f Medical Examiners Comprehensive Basic Science Self-Assessment

Tim e Remaining: 3 hr 9 min 49 sec

37. A 20-year-old woman comes to the physician for preconceptional counseling because she is concerned that she will not be able to conceive children. She has never had a menstrual period. She has normal female body habitus. Serum follicle-stimulating hormone concentration is 120 mlU/mL. Her karyotype is 45,X. In counseling this patient, which of the following should be recommended? o A) Oocyte donation O B) Ovulation induction O C) Surrogacy with her eggs O D) Timed intercourse

Previous

Lab Values

Review

Help
-

Pause

O
.,il,

4:04 PM
< >

04/08/2012

[^p National Board of Medical Examiners - Google Chrom e Q https://www.starttest.com/api/5.1.1.0/TTDStart.aspx?SVC=lbfd4732-460a-454b-90b5-9985c0560007

I'

Exam Section 2: Item 38 o f 50 I Mark

National Board o f Medical Examiners Comprehensive Basic Science Self-Assessment

Tim e Remaining: 3 hr 9 min 17 sec

38. Alveolar ventilation and C 0 2 production double during moderate exercise in a 48-year-old man. Which of the following best describes the effect on arterial Pco2? O A) Divided by 4 O B) Divided by 2 o C) Not changed D) Doubled E) Quadrupled

Previous

Lab Values

Review

Help -

Pause

O
..in

< >

4:04 PM 04/08/2012

National Board of Medical Examiners - Google Chrom e g https://www.starttest.eom/api/5.1.1.0/TTDStart.aspx?SVC=lbfd4732-460a-454b-90b5-9985c0560007

Exam Section 2: Item 39 o f 50 0 Mark

National Board o f Medical Examiners Comprehensive Basic Science Self-Assessment

Tim e Remaining: 3 hr 8 min 12 sec

39. A 56-year-old woman with hypertension has Raynaud disease. Which of the following anti hypertensive drugs is most beneficial for both disorders? O A) Captopril O B)Clonidine O C) Losartan o D) Nifedipine O E) Propranolol

National Board of Medical Examiners - Google Chrom e Q https://www.starttest.com/api/5.1.1.0/ITDStart.aspx?SVC=lbfd4732-460a-454b-90b5-9985c0560007

I___

Exam Section 2: Item 40 o f 50 I Mark

National Board o f Medical Examiners Comprehensive Basic Science Self-Assessment

Tim e Remaining: 3 hr 6 min 59 sec

40. A 1-year-old boy is brought to the emergency department because of a 6-day history of temperatures to 39.4C (103F) and a 2-day history of a severe diaper rash and swelling of his hands and feet. His temperature is now 39.4C (103F). A photograph of the genital area is shown. The face and lips appear red and the conjunctivae appear injected. There is bilateral cervical lymphadenopathy. The lungs are clear to auscultation. Cardiac examination shows an S 3 gallop with no murmur. There is edema and erythema of the hands and feet. Which of the following pathologic findings is most likely in this patient? o A) Acute arteritis with aneurysms in coronary arteries B) Epidermal hyperplasia with epidermal microabscesses and parakeratosis C) Granulomas with caseous necrosis in cervical lymph nodes D) Granulomatous arteritis in cervical and temporal arteries E) Paracortical lymphoid hyperplasia with eosinophilic intranuclear inclusions in perihilar lymph nodes

[^f National Board of Medical Examiners - Google Chrom e

Q https://www.starttest.com/api/5.1.1.0/TTDStart.aspx?SVC=lbfd4732-460a-454b-90b5-9985c0560007 Exam Section 2: Item 41 o f 50 Q Mark National Board o f Medical Examiners Comprehensive Basic Science Self-Assessment Tim e Remaining: 3 hr 4 min 19 sec

41. A 26-year-old man comes to the physician because of fever, cough, chest pain, and malaise for 2 weeks. He moved to central California 6 months ago. A complete blood count shows mild eosinophilia. A chest x-ray shows patchy bronchopneumonia. Culture of the sputum grows a mold with the morphology shown in the photomicrograph, with the arrow indicating the infectious particle. Which of the following is the most likely causal organism? A)

Actinomyces israelii

B) Coccidioides immitis
C) Histoplasma capsulatum D) Legionella pneumophila E) Mycobacterium tuberculosis F) Nocardia brasiliensis G) Staphylococcus aureus H) Streptococcus pneumoniae

Previous

Lab Values

Review

Pause
4:10 PM 04/08/2012

National Board of Medical Examiners - Google Chrom e

|. C = 3

| 0

https://www.starttest.com/api/5.1.1.0/ITDStart.aspx?SVC=lbfd4732-460a-454b-90b5-9985c0560007

Exam Section 2: Item 42 o f 50 H Mark

National Board o f Medical Examiners Comprehensive Basic Science Self-Assessment

Tim e Remaining: 3 hr 2 min 27 sec

42. A 37-year-old man comes to the physician because of progressive weakness and diarrhea during the past 2 months. He has had a 15-kg (33-lb) weight loss due to a decrease in appetite during this period. His blood pressure is 65/40 mm Hg. Physical examination shows hyperpigmentation of the skin. Laboratory studies show hyponatremia and hyperkalemia. Hypotension and electrolyte disturbances in this patient are most likely due to the absence of a hormone that is primarily synthesized in which of the following labeled regions in the photomicrograph of the organ shown?

Previous

Lab Values

Review

Pause

C l
4:11 PM 04/08/2012

https://www.starttest.com/api/5.1.1.0/TTDStart.aspx?SVC=lbfd4732-460a-454b-90b5-9985c0560007

Exam Section 2: Item 43 o f 50 H Mark

National Board o f Medical Examiners Comprehensive Basic Science Self-Assessment

Tim e Remaining: 2 hr 58 min 28 sec

43. A previously healthy 55-year-old woman comes to the physician because of the gradual onset of fever, fatigue, and pain in her muscles and joints during the past 3 weeks; she has had a 3.6-kg (8-lb) weight loss during this period. She does not smoke cigarettes, drink alcohol, or use illicit drugs. She takes no medications. Her temperature is 38.2C (100.8F), pulse is 90/min, and blood pressure is 140/95 mm Hg. Examination of the trunk and extremities shows areas of raised, reticular, cyanotic discoloration consistent with livedo reticularis. There is left footdrop. Laboratory studies show: Erythrocyte sedimentation rate Urine Blood Protein RBC casts 110 mm/h 2+ 2+ present

Serum test results are positive for perinuclear anti neutrophil cytoplasmic antibodies. Which of the following is the most likely diagnosis? A) Allergic interstitial nephritis B) Antiphospholipid antibody syndrome O C) IgA nephropathy O D) Occult neoplasm o E) Vasculitis

LI

National Board of Medical Examiners - Google Chrom e Q https://www.starttest.com/api/5.1.1.0/ITDStart.aspx?SVC=lbfd4732-460a-454b-90b5-9985c0560007

I___

Exam Section 2: Item 44 o f 50 I Mark

National Board o f Medical Examiners Comprehensive Basic Science Self-Assessment

Tim e Remaining: 2 hr 57 min 14 sec

44. A 5-year-old girl is brought to the physician by her parents because she is developing breasts. Examination shows breast enlargement and scant pubic hair. X-rays of the left hand show a bone age of 9 years. An intravenous bolus of gonadotropin-releasing hormone induces a marked increase in serum luteinizing hormone concentration 1 hour later. Which of the following drugs is most appropriate to suppress premature puberty in this patient? O A) ACTH O B) Hydrocortisone C) Ketoconazole o D) Leuprolide O E) Progesterone O F) Spironolactone O G) Tamoxifen O H) Testosterone

National Board of Medical Examiners - Google Chrom e Q https://www.starttest.com/api/5.1.1.0/ITDStart.aspx?SVC=lbfd4732-460a-454b-90b5-9985c0560007

I___

Exam Section 2: Item 45 o f 50 I Mark

National Board o f Medical Examiners Comprehensive Basic Science Self-Assessment

Tim e Remaining: 2 hr 55 min 3 sec

45. A 27-year-old man is brought to the emergency department by his mother 30 minutes after a generalized tonic-clonic seizure. She reports that he is now becoming more arousable. He has schizophrenia, and his mother says that he stopped taking his haloperidol 3 months ago. He has been drinking a lot of water during the past 2 months to "rid himself of toxins." On arrival, he is initially unresponsive but then becomes arousable. His temperature is 37.8C (100F), pulse is 90/min and regular, and blood pressure is 110/78 mm Hg. Examination shows normal skin turgor. The neck is supple with normal range of motion. Cardiopulmonary examination shows no abnormalities. Abdominal examination shows no abnormalities. There is no peripheral edema. Laboratory studies show: Serum Na+ K+ Clh c o 3-

Urea nitrogen Creatinine Urine specific gravity

116 mEq/L 4.2 mEq/L 80 mEq/L 18 mEq/L 8 mg/dL 0.8 mg/dL 1.002

Urine dipstick shows no other abnormalities. Results of additional laboratory studies are pending. Which of the following is the most likely cause of this patient's hyponatremia? O A) Adverse effect of haloperidol B) Hypothyroidism o C) Psychogenic polydipsia D) Syndrome of inappropriate secretion of ADH (vasopressin) O E) Tumor in the central nervous system

National Board of Medical Examiners - Google Chrom e Q https://www.starttest.com/api/5.1.1.0/ITDStart.aspx?SVC=lbfd4732-460a-454b-90b5-9985c0560007

I___

Exam Section 2: Item 46 o f 50 0 Mark

National Board o f Medical Examiners Comprehensive Basic Science Self-Assessment

Tim e Remaining: 2 hr 53 min 27 sec

46.

A study is conducted of the oxygen-binding properties of hemoglobin in an experimental animal. The hydrogen ion concentration is increased in blood. Results show that the oxygenhemoglobin dissociation curve shifts to the right. This effect will most likely result in which of the following processes? A) Binding of carbon dioxide in tissue capillaries

O B) Binding of oxygen in the lungs C) Decrease in erythrocyte 2,3-bisphosphoglycerate (2,3-BPG) concentration D) Increase in erythrocyte 2,3-BPG concentration E) Release of carbon dioxide in the lungs o F) Release of oxygen in tissue capillaries

National Board of Medical Examiners - Google Chrom e Q https://www.starttest.com/api/5.1.1.0/ITDStart.aspx?SVC=lbfd4732-460a-454b-90b5-9985c0560007

Exam Section 2: Item 47 o f 50 I Mark

National Board o f Medical Examiners Comprehensive Basic Science Self-Assessment

Tim e Remaining: 2 hr 51 min 42 sec

47. A 20-year-old man with mental retardation is brought to the physician for a routine examination. Physical examination shows a long face, prominent ears, and large testes. His younger brother and a maternal uncle have similar features. Analysis of the patient's DNA shows 800 CGG repeated sequences (N<60) in the 5' untranslated region of the FM R 1 gene. This region of expanded CGG trinucleotide repeats is heavily methylated. Which of the following is the most likely effect of these expanded nucleotide repeats on transcription of FM R1 mRNA? C A) Alteration of mRNA splicing B) Decreased transcription C) Enhancement of mRNA degradation o D) Incorporation of CGG repeats into mRNA E) Increased binding of RNA polymerase

National Board of Medical Examiners - Google Chrom e Q https://www.starttest.com/api/5.1.1.0/ITDStart.aspx?SVC=lbfd4732-460a-454b-90b5-9985c0560007

I___

Exam Section 2: Item 48 o f 50 I Mark

National Board o f Medical Examiners Comprehensive Basic Science Self-Assessment

Tim e Remaining: 2 hr 50 min 33 sec

48. A 54-year-old woman undergoes a biopsy of a 5-cm coin lesion in the right upper lobe of the lung. Examination of the biopsy specimen shows a proliferation of irregularly shaped glands containing cells with hyperchromatic and pleomorphic nuclei invading the pulmonary parenchyma and lymphovascular spaces. Which of the following best describes this patient's pulmonary lesion? o A) Adenocarcinoma O B) Carcinoid tumor C) Hamartoma D) Small cell carcinoma E) Squamous cell carcinoma

[^p National Board of Medical Examiners - Google Chrom e Q https://ww w .starttest.eom /api/5.l.1.0/TTDStart.aspx?SVC=lbfd4732-460a-454b-90b5-9985c0560007

Exam Section 2: Item 49 o f 50 0 Mark

National Board o f Medical Examiners Comprehensive Basic Science Self-Assessment

Tim e Remaining: 2 hr 48 min 4 sec

49. A healthy 40-year-old man comes to the physician requesting a medication to help him sleep on an overnight flight to Australia. Physical examination shows no abnormalities. Which of the following is the most appropriate pharmacotherapy for this patient? O A) Bupropion O B) Buspirone C) Clonazepam O D) Flurazepam o E) Zolpidem

Previous

Next

H r %

Lab Values

Review

Help

Pause

&

..h i

$ i >

4:27 PM
04/08/2012

National Board of Medical Examiners - Google Chrom e Q https://www.starttest.com/api/5.1.1.0/ITDStart.aspx?SVC=lbfd4732-460a-454b-90b5-9985c0560007

I___

Exam Section 2: Item 50 o f 50 I Mark

National Board o f Medical Examiners Comprehensive Basic Science Self-Assessment

Tim e Remaining: 2 hr 46 min 27 sec

50. An 18-year-old man comes to the physician because of nausea, headache, blood in his urine, and malaise for 3 days. Three weeks ago, he had severe pharyngitis that resolved spontaneously after several days without antibiotic therapy. His blood pressure is 165/88 mm Hg. Physical examination shows mild peripheral edema. His serum creatinine concentration is 2.1 mg/dl_, and serum CH50 is markedly decreased. Urinalysis shows: Protein RBC WBC RBC casts Blood cultures are negative. Which of the following is the most likely causal organism? A) Streptococcus agalactiae (group B) B) S. mutans O C) S. pneumoniae 2+ 25 50/hpf 10-20/hpf present

D) S. pyogenes (group A) O E) Viridans streptococci

National Board of Medical Examiners - Google Chrom e Q https: 7 www.starttest.com/api/5.1.1.0/rrDStart.aspx?SVC=lbfd4732-460a-454b-90b5-9985c0560007

Exam Section 3: Item 1 o f 50 I Mark

National Board o f Medical Examiners Comprehensive Basic Science Self-Assessment

Tim e Remaining: 4 hr 15 min 50 sec

1. A 22-year-old woman volunteers for a phase 1 clinical trial of the pharmacokinetics of a new drug. She receives a single intravenous bolus of the drug in her left upper extremity. Blood samples are then drawn at different times from her right upper extremity. The concentration of the drug is measured by a specific assay, and the results are shown (log of plasma concentration [C J versus time). Which of the following best explains the two-phase results obtained? A) Normal elimination followed by saturation of liver enzymes B) Rapid absorption followed by binding to plasma proteins C) Rapid absorption followed by slower elimination o D) Rapid distribution to tissues followed by normal elimination E) Rapid excretion by kidneys followed by metabolism in liver

Time

Lab Values

Review

Pause

C l
4:46 PM 04/08/2012

National Board of Medical Examiners - Google Chrom e Q https://www.starttest.com/api/5.1.1.0/ITDStart.aspx?SVC=lbfd4732-460a-454b-90b5-9985c0560007

I___ I
National Board o f Medical Examiners Comprehensive Basic Science Self-Assessment Tim e Remaining: 4 hr 12 min 39 sec

Exam Section 3: Item 2 o f 50 I Mark

2. A 3-week-old newborn is brought to the physician because of recurrent vomiting after feeding since birth. The mother says that her child is eager to feed even after vomiting. There is no fever. Abdominal examination shows a firm, 1- to 2-cm, mobile mass in the epigastrium to the right of the midline. If this condition has a lower threshold of liability in males than in females, which of the following relatives of this patient is at greatest risk for also developing this disorder? o A) Brother, if newborn is female O B) Brother, if newborn is male C) Fraternal male twin, if newborn is male O D) Sister, if newborn is female O E) Sister, if newborn is male

National Board of Medical Examiners - Google Chrom e Q https://www.starttest.com/api/5.1.1.0/ITDStart.aspx?SVC=lbfd4732-460a-454b-90b5-9985c0560007

I___ I
National Board o f Medical Examiners Comprehensive Basic Science Self-Assessment Tim e Remaining: 4 hr 11 min 21 sec

Exam Section 3: Item 3 o f 50 I Mark

3. A 2-month-old boy is brought to the physician because of failure to thrive and poor feeding since birth. He was recently adopted from Romania and has been fed cow's milk. He is below the 5th percentile for length and weight. Physical examination shows jaundice, cataracts, and hepatomegaly. Serum studies show a decreased glucose concentration. The urine shows a positive reaction to a copper reduction test and a negative reaction to a test agent that contains glucose oxidase. Deficiency of which of the following hepatic enzymes is the most likely cause of the disorder in this patient? O A) Fructokinase B) Fructose-1,6-bisphosphate aldolase C) Galactokinase o D) Galactose-1-phosphate uridyltransferase E) Glucose-6-phosphatase

National Board of Medical Examiners - Google Chrom e

Q https://www.starttest.eom/api/5.l.1.0/TTDStart.aspx?SVC=lbfd4732-460a-454b-90b5-9985c0560007 Exam Section 3: Item 4 o f 50 H Mark National Board o f Medical Examiners Comprehensive Basic Science Self-Assessment

4. A 65-year-old man undergoes an operation to remove an abdominal mass. Histologic examination of the mass shows a large number of blood vessels. Further analysis shows an increased concentration of vascular endothelial growth factor (VEGF) in the tumor. Which of the following most likely stimulated the production of VEGF in the mass in this patient? o A) Decreased endostatin O B) Decreased Pco2 C) Decreased Po2 D) Decreased thrombospondin E) Increased endostatin O F) Increased Pco2 O G) Increased Po2 H) Increased thrombospondin

National Board of Medical Examiners - Google Chrom e Q https://www.starttest.com/api/5.1.1.0/ITDStart.aspx?SVC=lbfd4732-460a-454b-90b5-9985c0560007

I___ I
National Board o f Medical Examiners Comprehensive Basic Science Self-Assessment Tim e Remaining: 4 hr 8 min 35 sec

Exam Section 3: Item 5 o f 50 I Mark

5. A female newborn delivered at term is found to have a cleft palate with cleft lip bilaterally. Physical examination shows no other abnormalities. Cranial ultrasonography shows no abnormalities. Failure of fusion at which of the following sites is the primary cause of this patient's cleft lip? A) Frontonasal and eye prominences B) Frontonasal and maxillary prominences C) Mandibular process and external auditory meatus D) Mandibular prominence and secondary pharyngeal arch o E) Maxillary and nasal prominences

[^f National Board of Medical Examiners - Google Chrom e

| fay r

-|

Q https://www.starttest.com/api/5.1.1.0/TTDStart.aspx?SVC=lbfd4732-460a-454b-90b5-9985c0560007 Exam Section 3: Item 6 o f 50 H Mark National Board o f Medical Examiners Comprehensive Basic Science Self-Assessment Tim e Remaining: 4 hr 7 min 1 sec

6. A 57-year-old woman is brought to the emergency department because she has been unable to open her left eye since she awoke 2 hours ago. She has no history of trauma to the eye. Her vital signs are within normal limits. When the left eyelid is raised during examination, her eye is fixed in the out position. When the patient is asked to move her eye out, the eye intorts. An MRI of the brain shows an aneurysm of the left posterior communicating artery. The most likely cause of this patient's condition is compression of which of the following nerves? O A) Abducens nerve B) Long ciliary nerve o C) Oculomotor nerve D) Ophthalmic branch of the trigeminal nerve E) Trochlear nerve

Previous

p 1
Lab Values

Review -

Pause

C l
..in

< >

4:56 PM 04/08/2012

LI

National Board of Medical Examiners - Google Chrom e Q https://www.starttest.com/api/5.1.1.0/ITDStart.aspx?SVC=lbfd4732-460a-454b-90b5-9985c0560007

I__ I
National Board o f Medical Examiners Comprehensive Basic Science Self-Assessment Tim e Remaining: 4 hr 5 min 57 sec

Exam Section 3: Item 7 o f 50 I Mark

7. A 33-year-old woman has episodes of severe paroxysmal pain in the right cheek, lower eyelid, and upper lip. These episodes are triggered by light touch in the affected areas. The most likely diagnosis is neuralgia involving which of the following nerves? A) Buccal branch of the facial B) Glossopharyngeal C) Mandibular division of the trigeminal o D) Maxillary division of the trigeminal E) Ophthalmic division of the trigeminal F) Zygomatic branch of the facial

https://www.starttest.com/api/5.1.1.0/TTDStart.aspx?SVC=lbfd4732-460a-454b-90b5-9985c0560007

Exam Section 3: Item 8 o f 50 H Mark

National Board o f Medical Examiners Comprehensive Basic Science Self-Assessment

Tim e Remaining: 4 hr 3 min 17 sec

8. During an experiment, drug X is added to a muscle bath containing vascular smooth muscle precontracted to 5 grams (g) tension. The effects of drug X and other drugs on the force of muscle contraction (g) are shown in the table.

Standard Drugs
Drug vehicle Norepinephrine Isoproterenol Phenylephrine Which of the following drugs is most likely to produce effects similar to those of drug X? O A) Albuterol O B) Atropine C) Methoxamine o D) Prazosin O E) Propranolol

Force Before Drug X (g)


5.0 15.1 0.5 13.0

Force After Drug X (g)


4.9

6.8 0.6
5.5

National Board of Medical Examiners - Google Chrom e Q https://wvw.starttest.com /api/511M TDStart.aspx7SVC3lbfd4732-460a-454b-90b5-9985c0560007

I__ I
National Board o f Medical Examiners Comprehensive Basic Science Self-Assessment Tim e Remaining: 3 hr 56 min 56 sec

Exam Section 3: Item 9 o f 50 0 Mark

9. A 5-year-old boy with asthma is brought to the physician because of a 3-month history of recurrent cough, intermittent wheezing, and difficulty breathing. He has been admitted to the hospital twice for pneumonia during the past 12 months. He is at the 60th percentile for height and the 50th percentile for w eight His respirations are 25/min. Wheezing and crackles are heard between only the fourth to the sixth intercostal spaces on the right side of the chest. Diminished tactile fremitus and dullness are present over the right anterior section of the chest between the 4th and 6th ribs. Results of a PPD test are negative. A lateral chest x-ray shows a wedge-shaped density extending anteriorly and inferiorly from the hilum. A CT scan of the chest is most likely to show obstruction of which of the following bronchi? C A) Left main-stem B) Left upper lobe o C) Right lower lobe D) Right main-stem = > E) Right middle lobe

National Board of Medical Examiners - Google Chrom e Q https://wvw.starttest.com /api/511M TDStart.aspx7SVC3lbfd4732-460a-454b-90b5-9985c0560007

I___ I
National Board o f Medical Examiners Comprehensive Basic Science Self-Assessment Tim e Remaining: 3 hr 56 min 19 sec

Exam Section 3: Item 10 o f 50 I Mark

10. A 6-year-old girl is brought to the physician by her mother because of swollen, itchy eyes, a runny nose, and sneezing for the past week. Knowledge of which of the following components of the patient's history is likely to be most productive in establishing a diagnosis? O A) New medications o B) New pet in the household C) Occupations of parents D) Recent illness in a family member O E) Recent school performance

[^p National Board of Medical Examiners - Google Chrom e Q https://ww w .starttest.eom /api/5.l.1.0/TTDStart.aspx?SVC=lbfd4732-460a-454b-90b5-9985c0560007

I'

Exam Section 3: Item 11 o f 50 I Mark

National Board o f Medical Examiners Comprehensive Basic Science Self-Assessment

Tim e Remaining: 3 hr 55 min 51 sec

11.

A 21 -year-old woman with primary pulmonary hypertension begins treatment with bosentan. As a result, blockade of which of the following is most likely to occur?

O A) Angiotensin II receptors B) Calcium channels o C) Endothelin receptors O D) Production of phosphodiesterase 5 E)Voltage-gated sodium channels

Previous

Lab Values

Review

Help
-

Pause

O
.,il,

5:07 PM
< >

04/08/2012

National Board of Medical Examiners - Google Chrom e Q https://www.starttest.com/api/5.1.1.0/ITDStart.aspx?SVC=lbfd4732-460a-454b-90b5-9985c0560007

I___ I
National Board o f Medical Examiners Comprehensive Basic Science Self-Assessment Tim e Remaining: 3 hr 53 min 37 sec

Exam Section 3: Item 12 o f 50 0 Mark

12.

A 30-year-old woman comes to the physician for an initial evaluation. She has a history of illicit intravenous drug use and is now completing a methadone maintenance program. Physical examination shows scars in the antecubital fossae. A chest x-ray shows small nodules in the perihilar lung fields. A biopsy specimen from the perihilar regions will most likely show small foreign particles surrounded by which of the following? A) C) Granulation tissue Hemosiderin-laden macrophages

o B) Granulomatous inflammation O D) Neutrophilic abscess E)Proliferated capillaries

National Board of Medical Examiners - Google Chrom e Q https://www.starttest.com/api/5.1.1.0/ITDStart.aspx?SVC=lbfd4732-460a-454b-90b5-9985c0560007

I___ I
National Board o f Medical Examiners Comprehensive Basic Science Self-Assessment Tim e Remaining: 3 hr 51 min 34 sec

Exam Section 3: Item 13 o f 50 I Mark

13. Four children, ages 2 through 6 years, are brought to the physician by their mother because of a 2-week history of irritability and forgetfulness. Their mother says, "Their behavior is the last straw. They fight all the time and copy each other's bad behavior. They are also lazy; when theyre not fighting, theyre sleeping. And when I ask the older ones to do their chores, they whine and say they can't because they have a headache or they're dizzy." She adds, "I barely have enough money to buy food, and I've been trying to avoid having our electricity turned off by using my portable heater." Which of the following initial actions by the physician is most appropriate? A) C) Offer the mother a behavior program with the use of time-out for her children Suggest that the mother consider medication for her own stress level

O B) Review the adequacy of the children's diet o D) Assess the possibility of carbon monoxide toxicity O E) Call child protective services

https://www.starttest.com/api/5.1.1.0/TTDStart.aspx?SVC=lbfd4732-460a-454b-90b5-9985c0560007

Exam Section 3: Item 14 o f 50 I Mark

National Board o f Medical Examiners Comprehensive Basic Science Self-Assessment

Tim e Remaining: 3 hr 50 min 2 sec

14.

A 55-year-old woman comes to the physician because of a 3-month history of increased fatigue and difficulty sleeping. She says that she feels sad and empty and is not interested in her hobbies anymore. She has no history of major medical illness. She takes no medications. She does not smoke cigarettes or drink alcohol. Physical examination and laboratory studies show no abnormalities. Treatment with escitalopram is started. The next day, the patient's husband calls the physician to ask about his wife's diagnosis. Which of the following is the most appropriate response by the physician to the patient's husband?

O A) "I can tell you about your wife's diagnosis since you are her legal next of kin." B) "I cannot discuss any patient information over the telephone, so I recommend that you come to the office with your wife instead." C) "I can't disclose your wife's diagnosis to you unless you have health care power-of-attorney rights." o D) "I'm sorry, but I cannot tell you anything about your wife's diagnosis without her permission." E) "Since I don't know the source of your wife's depression, you should discuss this with her."

National Board of Medical Examiners - Google Chrom e Q https://www.starttest.com/api/5.1.1.0/ITDStart.aspx?SVC=lbfd4732-460a-454b-90b5-9985c0560007

I___ I
National Board o f Medical Examiners Comprehensive Basic Science Self-Assessment Tim e Remaining: 3 hr 49 min 12 sec

Exam Section 3: Item 15 o f 50 I Mark

15. A 37-year-old woman has a 6-month history of increasing pain, stiffness, and swelling in her hands and wrists. Ibuprofen alleviates her symptoms for brief periods. Examination shows swelling of the soft tissue of the wrists and the metacarpophalangeal and proximal interphalangeal joints bilaterally. Which of the following is the most likely cause of these symptoms? A) Cartilage degeneration B) Deposition of uric acid crystals O C) Infection o D) Systemic inflammatory disease

0 E) Trauma

National Board of Medical Examiners - Google Chrom e Q https://www.starttest.com/api/5.1.1.0/ITDStart.aspx?SVC=lbfd4732-460a-454b-90b5-9985c0560007

Exam Section 3: Item 16 o f 50 0 Mark

National Board o f Medical Examiners Comprehensive Basic Science Self-Assessment

Tim e Remaining: 3 hr 47 min 20 sec

16.

Protection against paralytic poliomyelitis can be achieved either by oral administration of live attenuated poliovirus (Sabin) vaccine or by parenteral administration of killed poliovirus (Salk) vaccine. The common features of these two vaccines, which accounts for their efficacy, is their ability to induce which of the following poliovirus-specific immune responses? A) Activated CD8+ cytotoxic T lymphocyte effectors in the circulation

O B) Activated CD8+ cytotoxic T lymphocyte effectors in the gut C) CD8+ memory T lymphocytes D) Neutralizing antibodies in the circulation @ > E) Neutralizing secretory IgA antibodies in the gut

[^p National Board of Medical Examiners - Google Chrom e Q https://ww w .starttest.eom /api/5.l.1.0/TTDStart.aspx?SVC=lbfd4732-460a-454b-90b5-9985c0560007

Exam Section 3: Item 17 o f 50 I Mark

National Board o f Medical Examiners Comprehensive Basic Science Self-Assessment

Tim e Remaining: 3 hr 46 min 10 sec

17.

A 3-year-old girl is brought to the emergency department because of nausea and vomiting that began after she ingested a large quantity of her grandfather's "heart pills." An ECG shows a third-degree atrioventricular nodal block with ventricular escape rhythm that resolves after the administration of atropine. Which of the following agents was most likely ingested? A) Captopril

o B) Digoxin C) Hydrochlorothiazide D) Isosorbide dinitrate

Previous

Lab Values

Review

Help -

Pause

O
..in

< >

5:17 PM 04/08/2012

National Board of Medical Examiners - Google Chrom e Q https://www.starttest.com/api/5.1.1.0/ITDStart.aspx?SVC=lbfd4732-460a-454b-90b5-9985c0560007

I___ I
National Board o f Medical Examiners Comprehensive Basic Science Self-Assessment Tim e Remaining: 3 hr 40 min 31 sec

Exam Section 3: Item 18 o f 50 0 Mark

18. A previously unrecognized, rapidly progressive degenerative neurologic illness is detected in the population of a small island in the Pacific Ocean. Autopsies are performed on several patients who died of the disease. Electron microscopy of infected neurons shows spherical to conical viral particles. Brain tissue is then used to inoculate mice and a variety of cell lines, but no cytopathic effect is observed. Further enzymatic studies show that infected cells contained RNA-dependent DNA polymerase activity. The virus causing this infection is most closely related to which of the following viruses? O A) Coxsackievirus O B) Herpes simplex virus

m C) HIV D) Human papillomavirus O E) West Nile virus

National Board of Medical Examiners - Google Chrom e Q https://www.starttest.com/api/5.1.1.0/ITDStart.aspx?SVC=lbfd4732-460a-454b-90b5-9985c0560007

I___ I
National Board o f Medical Examiners Comprehensive Basic Science Self-Assessment Tim e Remaining: 3 hr 38 min 36 sec

Exam Section 3: Item 19 o f 50 I Mark

19.

During an evaluation for renal lithiasis, a 46-year-old man undergoes a series of imaging studies. An x-ray shows evidence of subcortical bone resorption. A dual energy x-ray absorptiometry scan shows decreased bone density. Laboratory studies show hypercalcemia and an increased parathyroid hormone concentration. A parathyroidectomy is recommended. During the operation, two parathyroid glands are found posterior to the thyroid gland on the left side, but only one, which appears to be the inferior, is present on the right. Because of the similarity of its developmental origin, which of the following additional organs should be explored to find the remaining parathyroid?

O A) Palatine tonsil O B) Parotid gland C) Sublingual gland D) Submandibular gland o E) Thymus

https://www.starttest.com/api/5.1.1.0/TTDStart.aspx?SVC=lbfd4732-460a-454b-90b5-9985c0560007

Exam Section 3: Item 20 o f 50 0 Mark

National Board o f Medical Examiners Comprehensive Basic Science Self-Assessment

Tim e Remaining: 3 hr 34 min 36 sec

20. A 12-year-old boy is admitted to the hospital because of lethargy, hip pain, and a temperature of 39.4C (103F). He has been hospitalized several other times because of pneumonia. His neonatal period was normal. Complete blood counts are within normal limits, and a test for HIV antibody is negative. Blood cultures grow Staphylococcus aureus. Serum

Next

Lab Values

Review

Pause

C l

https://www.starttest.com/api/5.1.1.0/TTDStart.aspx?SVC=lbfd4732-460a-454b-90b5-9985c0560007

Exam Section 3: Item 21 o f 50 H Mark

National Board o f Medical Examiners Comprehensive Basic Science Self-Assessment

Tim e Remaining: 3 hr 31 min 26 sec

21. A 5-year-old girl is brought to a new physician by her mother for an examination prior to attending kindergarten. Her mother says, "My daughterhas a murmur, but I was told that she'd grow out of it." The patient has no history of major medical illness. She is at the 50thpercentile for height and weight. A grade 2/6 systolicmurmur is heard over the left sternal border. Cardiac catheterization shows:

Location
Aorta Vena cava Pulmonary artery Right atrium Left atrium Right ventricle Left ventricle

Pressure (mm Hg)


105/60 7/2 25/9 4 8 25/4 105/8

0 2 Saturation
98% 75% 85% 75% 98% 83% 98%

The failure of which of the following during embryonic development best explains this patient's condition? O A) Closure of the ductus venosus B) Closure of the foramen ovale C) Development of the septum secundum o D) Fusion of the interventricular septum with endocardial cushions O E) Reabsorption of the septum primum

LI

National Board of Medical Examiners - Google Chrom e Q https://wvwv.starttest.com/api/5.1.1.0/ITDStart.aspx?SVC=lbfd4732-460a-454b-90b5-9985c0560007

Exam Section 3: Item 22 o f 50 I Mark

National Board o f Medical Examiners Comprehensive Basic Science Self-Assessment

Tim e Remaining: 3 hr 29 min 4 sec

22. A 34-year-old woman gives birth to a newborn at 32 weeks' gestation with a small placenta. The newborn is found to have multiple anomalies, including bilateral simian creases and a small thorax. The newborn dies 2 hours after birth. Chromosomal analysis shows a karyotype of 69,XXY. DNA analysis indicates that two sets of chromosomes are maternal in origin and one is paternal. Studies from other triploid pregnancies have shown that triploidy due to two sets of paternal chromosomes has poorly developed embryonic tissue. Differing phenotypes dependent on parental origin are most compatible with which of the following genetic mechanisms? O A) Anticipation B) Expansion of trinucleotide repeats o C) Imprinting O D) Loss of heterozygosity O E) Pleiotropy

National Board of Medical Examiners - Google Chrom e Q https://www.starttest.com/api/5.1.1.0/ITDStart.aspx?SVC=lbfd4732-460a-454b-90b5-9985c0560007

I__ I
National Board o f Medical Examiners Comprehensive Basic Science Self-Assessment Tim e Remaining: 3 hr 24 min 49 sec

Exam Section 3: Item 23 o f 50 I Mark

23. A 16-year-old boy is brought to the emergency department by his mother because of a 3-day history of fever, chills, and a nonproductive cough. His temperature is 38.5C (101.3F). Physical examination shows several large ecchymoses and numerous petechiae. Laboratory studies show: Hemoglobin Hematocrit Leukocyte count Platelet count Prothrombin time Partial thromboplastin time Plasma fibrinogen 10.2 g/dL 33% 31,000/mm3 14,000/mm3 22 sec (INR=2.1) 51 sec 87 mg/dL (N=200-400)

A peripheral blood smear shows promyelocytes containing Auer rods. Which of the following best explains these findings? A) Acquired factor VIII (antihemophilic) inhibitor B) Biliary cholestasis C) Disseminated intravascular coagulation D) Hemophilia A E) Iron deficiency anemia o F) Vitamin K deficiency

National Board of Medical Examiners - Google Chrom e Q https://www.starttest.com/api/5.1.1.0/ITDStart.aspx?SVC=lbfd4732-460a-454b-90b5-9985c0560007

I__ I
National Board o f Medical Examiners Comprehensive Basic Science Self-Assessment Tim e Remaining: 3 hr 22 min 9 sec

Exam Section 3: Item 24 o f 50 0 Mark

24.

A 76-year-old woman comes to the physician because of severe back pain for the past 2 weeks. She has no history of smoking. She has been taking prednisone (30 mg/day) for rheumatoid arthritis for the past 6 months and has received hormone replacement therapy with estrogen and progesterone for the past 15 years. Calcium and vitamin D intake are adequate. X-rays of the spine show a vertebral fracture. Which of the following is the most likely cause of the fracture? A) Decreased bone formation due to decreased calcium absorption

o B) Decreased bone formation due to inhibition of osteoblast differentiation C) Increased bone resorption due to decreased calcium absorption D) Increased bone resorption due to decreased serum parathyroid hormone concentration O E) Increased bone resorption due to estrogen receptor defect

[^p National Board of Medical Examiners - Google Chrom e Q https://ww w .starttest.eom /api/5.l.1.0/TTDStart.aspx?SVC=lbfd4732-460a-454b-90b5-9985c0560007

Exam Section 3: Item 25 o f 50 I Mark

National Board o f Medical Examiners Comprehensive Basic Science Self-Assessment

Tim e Remaining: 3 hr 21 min 3 sec

25. A 58-year-old woman comes to the physician because of a 3-week history of abdominal swelling and fatigue. Abdominal examination shows ascites. A left ovarian mass is found on pelvic examination. Microscopic examination of the ascitic fluid shows malignant cells. Additional malignant cells are most likely to be found in which of the following groups of lymph nodes? O A) Inferior mesenteric O B) Internal iliac o C) Para-aortic (lumbar) O D) Pararectal O E) Sacral

Previous

Lab Values

Review

Help

Pause

O
..in $ < >
5:42 PM 04/08/2012

National Board of Medical Examiners - Google Chrom e Q https://ww w .starttest.eom /api/5.l.1.0/TTDStart.aspx?SVC=lbfd4732-460a-454b-90b5-9985c0560007

IrO-UgM

Exam Section 3: Item 26 o f 50 I Mark

National Board o f Medical Examiners Comprehensive Basic Science Self-Assessment

Tim e Remaining: 3 hr 20 min 37 sec

26. An 80-year-old woman develops a bleeding duodenal ulcer after a 2-month stay in the intensive care unit. A decision is made to start the patient on a drug that will adhere strongly to the ulcer crater, thereby protecting the ulcer epithelium. Which of the following is the most appropriate pharmacotherapy? A) Calcium carbonate B) Magnesium hydroxide O C) Misoprostol O D) Ranitidine o E) Sucralfate

Previous

Lab Values

Review

Help -

Pause

O
..ill $ '< >
5:43 PM 04/08/2012

National Board of Medical Examiners - Google Chrom e Q https://www.starttest.com/api/5.1.1.0/ITDStart.aspx?SVC=lbfd4732-460a-454b-90b5-9985c0560007

I___ I
National Board o f Medical Examiners Comprehensive Basic Science Self-Assessment Tim e Remaining: 3 hr 19 min 20 sec

Exam Section 3: Item 27 o f 50 I Mark

27.

A 43-year-old woman comes to the physician because she is concerned about her risk for developing influenza. She is a nurse, and she recently cared for a man with influenza. She did not receive influenza virus vaccine this year. Physical examination shows no abnormalities. A drug that blocks which of the following viral enzymes is most appropriate to decrease this patient's risk for developing this infection?

O A) DNA polymerase B) Integrase o C) Neuraminidase D) Protease E) Reverse transcriptase

National Board of Medical Examiners - Google Chrom e Q https://www.starttest.com/api/5.1.1.0/ITDStart.aspx?SVC=lbfd4732-460a-454b-90b5-9985c0560007

I___ I
National Board o f Medical Examiners Comprehensive Basic Science Self-Assessment Tim e Remaining: 3 hr 18 min 8 sec

Exam Section 3: Item 28 o f 50 I Mark

28.

A previously healthy 2-year-old girl is brought to the physician because of three episodes of dark blood in her stool during the past week. Examination shows no masses in the abdomen or rectum. Test of the stool for occult blood is positive. A technetium 99m scan shows pooling of the radiolabel in the midabdomen. Which of the following is the most likely diagnosis?

O A) Anal fissure B) Appendicitis C) Congenital megacolon (Hirschsprung disease) D) Gastroesophageal reflux O E) Hemorrhoids o F) Meckel diverticulum

V ^ > National Board of Medical Examiners - Google Chrom e Q https://ww w .starttest.eom /api/5.l.1.0/TTDStart.aspx?SVC=lbfd4732-460a-454b-90b5-9985c0560007

I' C 3 | dpi

Exam Section 3: Item 29 o f 50 H Mark

National Board o f Medical Examiners Comprehensive Basic Science Self-Assessment

Tim e Remaining: 3 hr 17 min 7 sec

29. A male newborn delivered at term has marked hypopigmentation with white hair, milky white skin, and blue irides. The most likely cause of this condition is a defective enzyme contained in which of the following organelles? A) Golgi complex

O B) Lysosomes o C) Melanosomes D) Mitochondria E) Nucleus

Previous

Lab Values

Review

Help -

Pause

O
..in

< >

5:47 PM 04/08/2012

National Board of Medical Examiners - Google Chrom e Q https://www.starttest.com/api/5.1.1.0/ITDStart.aspx?SVC=lbfd4732-460a-454b-90b5-9985c0560007

I___ I
National Board o f Medical Examiners Comprehensive Basic Science Self-Assessment Tim e Remaining: 3 hr 14 min 38 sec

Exam Section 3: Item 30 o f 50 0 Mark

30. An 18-year-old man is brought to the emergency department in rural Illinois because of fever, confusion, and right arm weakness for 8 hours. His symptoms began with tingling and numbness 2 days ago. His temperature is 40C (104 F). Physical examination shows no other abnormalities. A lumbar puncture is done. Examination of cerebrospinal fluid shows: Glucose Protein Leukocyte count Lymphocytes Erythrocyte count 58 mg/dL 50 mg/dL 23/mm3 93% 3/mm3

An MRI of the brain shows no abnormalities. He is oriented to person but not to place or time. During the next 4 days his confusion progresses, and he develops hypersalivation and respiratoryfailure. He continues to deteriorate despite intubation. He dies four days later. Which of the following animals is the most likely source of this patient's infection? o A) Bat B) Cat C) Cow O D) Dog E) Sheep

https://www.starttest.com/api/5.1.1.0/TTDStart.aspx?SVC=lbfd4732-460a-454b-90b5-9985c0560007

Exam Section 3: Item 31 o f 50 0 Mark

National Board o f Medical Examiners Comprehensive Basic Science Self-Assessment

Tim e Remaining: 3 hr 12 min 32 sec

31.

A 12-year-old girl is brought to the physician because of a rash on her left buttock for the past 2 days. The rash developed after the family returned from a 2-week-long early summer vacation in Maine. Her vital signs are within normal limits. A photograph of the lesion is shown. The likely cause of this patient's infection is taxonomically and morphologically most similar to the infectious agent of which of the following conditions? A) Bacillary angiomatosis B) Chancroid C) Leptospirosis

C D) Lymphogranuloma venereum o E )Q fever

LI

https://www.starttest.com/api/5.1.1.0/TTDStart.aspx?SVC=lbfd4732-460a-454b-90b5-9985c0560007

Exam Section 3: Item 32 o f 50 H Mark

National Board o f Medical Examiners Comprehensive Basic Science Self-Assessment

Tim e Remaining: 3 hr 9 min 8 sec

32. A 15-month-old white boy is brought to the physician by his parents because of recurrent pneumonia since the age of 2 months and failure to thrive for 1 year. His stools are frequent and greasy. He has a 2-year-old cousin with a similar condition; his two older sisters are healthy. He is at the 10th percentile for length and below the 5th percentile for weight. Bilateral rhonchi are heard in both lungs. Examination of the stool shows an increased fat concentration. A chest x-ray shows increased bronchovascular markings bilaterally. There is no consanguinity. He is individual III-2 in the pedigree shown. Which of the following is the most likely explanation for the pattern of inheritance in this patient's family? A) Autosomal dominant with variable expressivity B) Autosomal dominant with variable penetrance o C) Autosomal recessive with high heterozygote frequency D) Autosomal recessive with low heterozygote frequency O E)X-linked rare gene

O
+

Unaffected fem ale U naffected m ale A ffected fem ale A ffected m ale

LI

National Board of Medical Examiners - Google Chrom e Q https://www.starttest.com/api/5.1.1.0/ITDStart.aspx?SVC=lbfd4732-460a-454b-90b5-9985c0560007

I__ I
National Board o f Medical Examiners Comprehensive Basic Science Self-Assessment Tim e Remaining: 3 hr 8 min 15 sec

Exam Section 3: Item 33 o f 50 I Mark

33.

A 50-year-old woman develops night sweats and a feeling of intense heat over the trunk and face. Pelvic examination shows atrophic vaginal mucosa but no other abnormalities. A Pap smear shows an increase in parabasal epithelial cells with no dysplasia. These symptoms are most likely due to decreased production of which of the following? A) Follicle-stimulating hormone by the pituitary B) Gonadotropin-releasing hormone by the hypothalamus C) Human chorionic gonadotropin production by thecal cells

O D) Luteinizing hormone by the pituitary o E) Steroid hormones by ovarian follicles

National Board of Medical Examiners - Google Chrom e Q https://www.starttest.com/api/5.1.1.0/ITDStart.aspx?SVC=lbfd4732-460a-454b-90b5-9985c0560007

I__ I
National Board o f Medical Examiners Comprehensive Basic Science Self-Assessment Tim e Remaining: 3 hr 6 min 57 sec

Exam Section 3: Item 34 o f 50 I Mark

34.

An investigator is studying oxygen transport in experimental animals. It is found that Po2in the renal vein is relatively high compared with venous Po2from most other organs. Which of the following best explains this finding? A) Counter-current multiplication in the kidneys allows them to use less oxygen for ion transport than other organs

O B) Po2 in the renal artery is greater than Po2 in other arteries o C) The ratio of oxygen consumption to blood flow is lower in the kidneys than in other organs D) Renal blood flow is a smaller fraction of the cardiac output than blood flow in other organs O E) Tissue Po2 is greater in every region of the kidneys

[^p National Board of Medical Examiners - Google Chrom e Q https://ww w .starttest.eom /api/5.l.1.0/TTDStart.aspx?SVC=lbfd4732-460a-454b-90b5-9985c0560007

Exam Section 3: Item 35 o f 50 I Mark

National Board o f Medical Examiners Comprehensive Basic Science Self-Assessment

Tim e Remaining: 3 hr 6 min 8 sec

35. A 52-year-old woman with type 2 diabetes mellitus has chronic renal failure. She takes no medications other than glipizide. Creatinine clearance is 20% of normal. Which of the following sets of laboratory findings in serum is most likely in this patient?

Ca2+
A) B) C) D) E) F) * *

Phosphate i

Parathyroid Hormone I

i i T i

I + I

Previous

Lab Values

Review

Help
-

Pause

O
.,il,

5:58 PM
< >

04/08/2012

National Board of Medical Examiners - Google Chrom e Q https://www.starttest.com/api/5.1.1.0/ITDStart.aspx?SVC=lbfd4732-460a-454b-90b5-9985c0560007

I___ I
National Board o f Medical Examiners Comprehensive Basic Science Self-Assessment Tim e Remaining: 3 hr 4 min 39 sec

Exam Section 3: Item 36 o f 50 I Mark

36. A previously healthy 20-year-old primigravid woman at 12 weeks' gestation comes to the physician 2 days after she noticed a painful swelling on the right side of her chest. Physical examination shows a 2 x 2-cm skin-colored mass with a nipple in the center on the chest wall just inferior to the right breast. If a biopsy specimen of the mass were obtained, it would most likely show a predominance of which of the following cell types? o A) Epithelial cells O B) Lymphocytes C) Macrophages D) Mast cells O E) Neutrophils

National Board of Medical Examiners - Google Chrom e Q https://www.starttest.com/api/5.1.1.0/ITDStart.aspx?SVC=lbfd4732-460a-454b-90b5-9985c0560007

Exam Section 3: Item 37 o f 50 I Mark

National Board o f Medical Examiners Comprehensive Basic Science Self-Assessment

Tim e Remaining: 3 hr 3 min 35 sec

37. A 28-year-old woman comes to the physician because of a 1-day history of shortness of breath and a 1-week history of fever, malaise, and a dry, nonproductive cough. Two months ago, she received an allogeneic bone marrow transplant that was complicated by acute graft-versus-host disease. Her temperature is 38.3C (101F) and respirations are 42/min. There is no rash. Generalized crackles are heard on auscultation of the chest. An x-ray of the chest shows bilateral interstitial infiltrates. A photomicrograph of tissue obtained at lung biopsy is shown. The most likely cause of her condition is infection with which of the following viruses? A) Adenovirus o B) Cytomegalovirus C) Enterovirus D) Influenza virus E) Measles virus F) Papillomavirus G) Parainfluenza virus H) Respiratory syncytial virus I) Rhinovirus J) Varicella-zoster virus

https://www.starttest.com/api/5.1.1.0/TTDStart.aspx?SVC=lbfd4732-460a-454b-90b5-9985c0560007

Exam Section 3: Item 38 o f 50 H Mark

National Board o f Medical Examiners Comprehensive Basic Science Self-Assessment

Tim e Remaining: 3 hr 0 min 35 sec

38. A 45-year-old woman comes to the physician because of a 2-month history of fatigue, nausea, and generalized bone pain. She has had a 2-kg (4.4-lb) weight loss during this period because of loss of appetite. She has a 4-month history of renal insufficiency. Her blood pressure is 170/100 mm Hg. Physical examination shows pallor and 2+ pitting edema of the feet. Laboratory studies show anemia. Her serum urea nitrogen concentration is 55 mg/dL, and serum creatinine concentration is 4 mg/dL. Bone x-rays show widened osteoid seams and subperiosteal erosions. Which of the following additional sets of serum findings is most likely in this patient?

Calcium
m A) B) C) D) E) F) G) * * *

Phosphate

Parathyroid Hormone +
i + I I

1,25-Dihydroxycholecalciferol

i i
^ .

i * *
^ ,

I I I
^ .

[^p National Board of Medical Examiners - Google Chrom e Q https://ww w .starttest.eom /api/5.l.1.0/TTDStart.aspx?SVC=lbfd4732-460a-454b-90b5-9985c0560007

Exam Section 3: Item 39 o f 50 I Mark

National Board o f Medical Examiners Comprehensive Basic Science Self-Assessment

Tim e Remaining: 2 hr 58 min 58 sec

39. A21-year-old man comes to the physician because of a 2-day history of fever, cough, and shortness of breath. He appears mildly ill. His temperature is 38.4C (101.1F), and respirations are 18/min. Diffuse crackles are heard on auscultation. A cold hemagglutinin test result is positive. A chest x-ray shows increased interstitial markings. The infectious agent is most likely susceptible to which of the following drugs? O A) Amoxicillin o B) Azithromycin O C) Ceftriaxone O D)Gentamicin E) Piperacillin

Previous

Lab Values

Review

Help

Pause

O
..in $ < >
6:05 PM 04/08/2012

National Board of Medical Examiners - Google Chrom e Q https://www.starttest.com/api/5.1.1.0/ITDStart.aspx?SVC=lbfd4732-460a-454b-90b5-9985c0560007

I__ I
National Board o f Medical Examiners Comprehensive Basic Science Self-Assessment Tim e Remaining: 2 hr 57 min 5 sec

Exam Section 3: Item 40 o f 50 I Mark

40. In an experimental animal, blood flow to a portion of the ventricular myocardium is 80% restricted by placing a ligature along a branch of the coronary artery. Thirty minutes later, blood flow is restored. Thirty minutes after the restoration of blood flow, the affected tissue is isolated for analysis. Histologic examination of the transiently ischemic region shows an increase in myocardial cell diameter compared with normal myocardium from the same heart and blebbing of the sarcolemma. The swelling of these cells is most directly attributable to which of the following changes? O A) Clumping of nuclear chromatin B) Decreased intracellular pH o C) Decreased sarcoplasmic ATP O D) Detachment of ribosomes O E) Exit of creatine kinase O F) Increased glycolysis G) Increased protein synthesis O H) Rupture of lysosomes

National Board of Medical Examiners - Google Chrom e Q https://www.starttest.com/api/5.1.1.0/ITDStart.aspx?SVC=lbfd4732-460a-454b-90b5-9985c0560007

I___ I
National Board o f Medical Examiners Comprehensive Basic Science Self-Assessment Tim e Remaining: 2 hr 55 min 22 sec

Exam Section 3: Item 41 o f 50 I Mark

41. A 6-month-old boy is brought to the physician by his mother because of a 10-week history of cold-like symptoms. The mother says that he tastes salty when she kisses him, and he has large, foul-smelling stools. There is no family history of a similar condition. He is at the 10th percentile for length and 3rd percentile for weight. Physical examination shows reduction in subcutaneous fat. Coarse rhonchi are heard bilaterally. Molecular analysis shows deletion of the Phe residue at position 508 in the cystic fibrosis transmembrane conductance regulator (CFTR). Which of the following most likely occurred in this patient as a result of this gene mutation? o A) Altered CFTR folding O B) 3-Amyloid configuration of CFTR C) Decreased surfactant protein synthesis O D) Increased CFTR conductance O E) Increased CFTR in the membrane

[^p National Board of Medical Examiners - Google Chrom e Q https://ww w .starttest.eom /api/5.l.1.0/TTDStart.aspx?SVC=lbfd4732-460a-454b-90b5-9985c0560007

Exam Section 3: Item 42 o f 50 I Mark

National Board o f Medical Examiners Comprehensive Basic Science Self-Assessment

Tim e Remaining: 2 hr 49 min 55 sec

42. In an epidemiologic study of workers in the aniline dye industry, 500 workers with bladder cancer and 200 workers without bladder cancer are selected. The investigators obtain a history of exposure to aniline dye in both groups of workers. Among workers with bladder cancer, 250 have a history of exposure to aniline dyes; among the 200 workers without bladder cancer, 50 have a history of exposure. Which of the following is the odds ratio for the exposure variable? A) 0.33 B) 0.5 C) 1.0 D) 1.5 E) 2.0 o F) 3.0

Previous

Lab Values

Review

Help

Pause

O
..in $ < >
6:23 PM 04/08/2012

National Board of Medical Examiners - Google Chrom e Q https://www.starttest.com/api/5.1.1.0/ITDStart.aspx?SVC=lbfd4732-460a-454b-90b5-9985c0560007

I__ I
National Board o f Medical Examiners Comprehensive Basic Science Self-Assessment Tim e Remaining: 2 hr 48 min 0 sec

Exam Section 3: Item 43 o f 50 I Mark

43. A 38-year-old man is scheduled to undergo a gastric reduction operation for treatment of morbid obesity. During the procedure to reduce stomach volume, a plastic band will be placed around the upper portion of the stomach. To encircle the stomach, the band will pass through which of the following structures? A) Hepatoduodenal ligament

o B) Lesser omentum O C) Phrenicocolic ligament D) Splenorenal ligament O E) Transverse mesocolon

National Board of Medical Examiners - Google Chrom e Q https://www.starttest.com/api/5.1.1.0/ITDStart.aspx?SVC=lbfd4732-460a-454b-90b5-9985c0560007

I___ I
National Board o f Medical Examiners Comprehensive Basic Science Self-Assessment Tim e Remaining: 2 hr 44 min 58 sec

Exam Section 3: Item 44 o f 50 I Mark

44. A 37-year-old woman comes to the physician for a follow-up examination. She has a 5-year history of cardiovascular disease and underwent an operation to insert two prosthetic valves 6 weeks ago. X-rays of the chest are shown. Which of the following valves was replaced by the prosthetic valve indicated by the arrows? A) Aortic B) Mitral o C) Pulmonic D) Tricuspid

National Board of Medical Examiners - Google Chrom e Q https://ww w .starttest.eom /api/5.l.1.0/TTDStart.aspx?SVC=lbfd4732-460a-454b-90b5-9985c0560007

IrO-UgM

Exam Section 3: Item 45 o f 50 I Mark

National Board o f Medical Examiners Comprehensive Basic Science Self-Assessment

Tim e Remaining: 2 hr 44 min 6 sec

45.

A previously healthy 4-year-old boy is brought to the physician because of pain in his right ear and irritability for the past 2 days. His temperature is 38.2C (100.8F). Examination of his right ear shows a red, opaque, bulging tympanic membrane. Which of the following is the most likely causal organism?

O A) Escherichia coli B) Mycobacterium tuberculosis C) Neisseria meningitidis D) Staphylococcus aureus E) Streptococcus pneumoniae

Previous

Next

Lab Values

Review

Help

Pause

..h i

$ i >

6:29 PM 04/08/2012

National Board of Medical Examiners - Google Chrom e g https://www.starttest.eom/api/5.1.1.0/TTDStart.aspx?SVC=lbfd4732-460a-454b-90b5-9985c0560007

Exam Section 3: Item 46 o f 50 I Mark

National Board o f Medical Examiners Comprehensive Basic Science Self-Assessment

Tim e Remaining: 2 hr 41 min 23 sec

46. A 32-year-old woman is found to have panic disorder with agoraphobia. A drug is prescribed that activates benzodiazepine binding sites on the Y-aminobutyric acidA (GABA a) receptor. This drug is most likely which of the following? o A) Alprazolam O B) Buspirone O C) Flumazenil D) Hydroxyzine O E) Ramelteon

National Board of Medical Examiners - Google Chrom e Q https://www.starttest.com/api/5.1.1.0/ITDStart.aspx?SVC=lbfd4732-460a-454b-90b5-9985c0560007

I___ I
National Board o f Medical Examiners Comprehensive Basic Science Self-Assessment Tim e Remaining: 2 hr 39 min 52 sec

Exam Section 3: Item 47 o f 50 I Mark

47.

A 24-year old man has had a purulent urethral discharge and dysuria for the past 2 days. Microscopic examination of a Gram stain of the urethral exudate shows gram-negative intracellular diplococci. He is treated with a standard dose of ceftriaxone. The symptoms disappear in 2 days but a mucoid discharge appears 10 days after treatment. Which of the following is the most likely explanation for the reappearance of symptoms? A) Ceftriaxone does not penetrate host cells

o B) The patient was initially infected with both Neisseria gonorrhoeae and Chlamydia trachomatis C) The patient was reinfected by an untreated sexual partner D) The patient's strain of Neisseria gonorrhoeae developed resistance to ceftriaxone

https://www.starttest.com/api/5.1.1.0/TTDStart.aspx?SVC=lbfd4732-460a-454b-90b5-9985c0560007

Exam Section 3: Item 48 o f 50 H Mark

National Board o f Medical Examiners Comprehensive Basic Science Self-Assessment

Tim e Remaining: 2 hr 38 min 37 sec

48. A 45-year-old man is brought to the emergency department 30 minutes after sustaining a gunshot wound to his abdomen. His blood pressure is 70/40 mm Hg. Physical examination shows pallor and diaphoresis. There is an entry wound in the mid-left region of the abdomen. His hematocrit is 20%. A CT scan of the abdomen is shown. An exploratory laparotomy shows that the abdominal cavity is filled with blood. Which of the following organs is most likely damaged in this patient? O A) Descending colon B) Left adrenal gland C) Left kidney o D) Spleen E) Tail of the pancreas

National Board of Medical Examiners - Google Chrom e Q https://www.starttest.com/api/5.1.1.0/ITDStart.aspx?SVC=lbfd4732-460a-454b-90b5-9985c0560007

Exam Section 3: Item 49 o f 50 I Mark

National Board o f Medical Examiners Comprehensive Basic Science Self-Assessment

Tim e Remaining: 2 hr 35 min 9 sec

49. A 31-year-old man comes to the physician because of chronic pain and decreased range of motion in the left knee. He runs 5 miles daily. Evaluation 2 months ago showed no structural abnormalities. The physician recommended physical therapy, use of nonsteroidal anti-inflammatory drugs (NSAIDs), and decreased running. The patient stopped physical therapy after 1week, rarely took NSAIDs, continued to run, and missed several appointments. Today he speaks with some sadness about the inadequacy of his pain control. Which of the following best describes the personality trait exhibited by this behavior? o A) Borderline C B) Dependent C) Histrionic C D) Obsessive-compulsive C E) Paranoid F) Passive-aggressive

National Board of Medical Examiners - Google Chrom e Q https://www.starttest.com/api/5.1.1.0/ITDStart.aspx?SVC=lbfd4732-460a-454b-90b5-9985c0560007

I___ I
National Board o f Medical Examiners Comprehensive Basic Science Self-Assessment Tim e Remaining: 2 hr 33 min 38 sec

Exam Section 3: Item 50 o f 50 I Mark

50. A 17-year-old African American boy is brought to the emergency department because of a 5-hour history of fever, progressive petechial rash, headache, and rigors. He was treated successfully for bacteremia due to Neisseria meningitidis 2 years ago. His temperature is 39.8C (103.6F), pulse is 120/min, respirations are 30/min, and blood pressure is 85/56 mm Hg. Physical examination shows evolving petechiae and purpura on the lower extremities. Treatment with empiric broad-spectrum antibiotics is begun. Twelve hours later, blood culture grows N. meningitidis. This patient should be examined for which of the following conditions? A) Agammaglobulinemia B) Chronic granulomatous disease o C) Complement deficiency D) HIV infection O E) Sickle cell disease

National Board of Medical Examiners - Google Chrom e Q https://wvw.starttest.com /api/511M TDStart.aspx7SVC3lbfd4732-460a-454b-90b5-9985c0560007

Exam Section 4: Item 1 o f 50 I Mark

National Board o f Medical Examiners Comprehensive Basic Science Self-Assessment

Tim e Remaining: 4 hr 17 min 38 sec

1. In immunocompromised patients, the case definition of a positive tuberculin skin test is changed from 10 mm of induration to 5 mm of induration. This change in case definition will have which of the following effects on incidence and prevalence of a positive tuberculin skin test?

Incidence
A) B) C) D) E)
o

Prevalence

T
< > < >

I I

T i

p 1
Lab Values Review
6:47 PM 04/08/2012

National Board of Medical Examiners - Google Chrom e Q https://www.starttest.com/api/5.1.1.0/ITDStart.aspx?SVC=lbfd4732-460a-454b-90b5-9985c0560007

I___ I
National Board o f Medical Examiners Comprehensive Basic Science Self-Assessment Tim e Remaining: 4 hr 15 min 52 sec

Exam Section 4: Item 2 o f 50 I Mark

2. A full-term 1-day-old male newborn undergoes hearing screening. The newborn has no visible malformations or abnormalities. Results of otoacoustic emissions testing are abnormal. A subsequent diagnostic brain stem auditory evoked response is also abnormal. Which of the following is the most compelling reason for this newborn screening program? A) Although congenital hearing loss occurs infrequently, screening is cost-effective B) Early diagnosis and treatment of hearing loss will prevent delay in motor development o C) Identification and treatment of hearing loss before the age of 6 months will allow for better prognosis of speech and language development D) Newborn screening allows for more time to prepare hearing aids so the newborn can be fitted with them when he or she reaches the age of 1 year E) Newborn screening will identify those children who will require a cochlear implant after the age of 5 years

https://www.starttest.com/api/5.1.1.0/TTDStart.aspx?SVC=lbfd4732-460a-454b-90b5-9985c0560007

Exam Section 4: Item 3 o f 50 H Mark

National Board o f Medical Examiners Comprehensive Basic Science Self-Assessment

Tim e Remaining: 4 hr 15 min 26 sec

3. A 12-month-old girl has a large extended family and is cared for by numerous relatives. According to her mother, the child does not spend much time on the floor because "somebody always seems to be carrying her around." Under these circumstances, this child is most likely to have problems with which of the following aspects of her development? O A) Cognitive B) Fine motor o C) Gross motor D)Language E) Social

National Board of Medical Examiners - Google Chrom e Q https://www.starttest.com/api/5.1.1.0/ITDStart.aspx?SVC=lbfd4732-460a-454b-90b5-9985c0560007

I__ I
National Board o f Medical Examiners Comprehensive Basic Science Self-Assessment Tim e Remaining: 4 hr 13 min 31 sec

Exam Section 4: Item 4 o f 50 I Mark

4. A 20-year-old man comes to the physician because of a 1-year history of progressive numbness of his arms. He recently burned his right thumb while cooking, but he felt no pain. Sensation to pain and temperature is decreased throughout both upper extremities and across the shoulders to the neck. Sensation is intact elsewhere. The most likely cause of his condition is a lesion in which of the following spinal cord structures? o A) Anterior white commissure O B) Dorsal column C) Intermediolateral cell column D) Lateral corticospinal tract E) Spinothalamic tract

National Board of Medical Examiners - Google Chrom e Q https://www.starttest.com/api/5.1.1.0/ITDStart.aspx?SVC=lbfd4732-460a-454b-90b5-9985c0560007

I__ I
National Board o f Medical Examiners Comprehensive Basic Science Self-Assessment Tim e Remaining: 4 hr 12 min 9 sec

Exam Section 4: Item 5 o f 50 I Mark

5. A 52-year-old man is brought to the emergency department 30 minutes after the onset of acute chest pain radiating to his back, shortness of breath, and weakness. He has a history of scoliosis. He is 208 cm (6 ft 10 in) tall and weighs 91 kg (200 lb); BMI is 21 kg/m2. His blood pressure is 90/60 mm Hg. Physical examination shows absence of pulses in the right upper extremity. Which of the following is the most likely diagnosis? o A) Aortic dissection B) Aortic occlusion C) Aortic stenosis D) Coarctation of the aorta E) Rheumatic aortitis

National Board of Medical Examiners - Google Chrom e Q https://www.starttest.com/api/5.1.1.0/ITDStart.aspx?SVC=lbfd4732-460a-454b-90b5-9985c0560007

I__ I
National Board o f Medical Examiners Comprehensive Basic Science Self-Assessment Tim e Remaining: 4 hr 10 min 16 sec

Exam Section 4: Item 6 o f 50 I Mark

6. A 24-year-old woman is brought to the emergency department 30 minutes after her father found her unresponsive on her bedroom floor. Her father says that the patient had been despondent for 2 weeks because of the ending of a long-term relationship. She had recently seen a physician seeking help for her emotional state. On arrival, she is unresponsive to painful stimuli. Her temperature is 35.7C (96.2F), pulse is 52/min, respirations are 5/min and shallow, and blood pressure is 85/55 mm Hg. Physical examination shows pinpoint pupils that are reactive to light. Neurologic examination shows no focal findings. This patient most likely has toxicity from which of the following? O A) Carbon monoxide B) Ethylene glycol C) Methanol o D) Opiates E) Selective serotonin reuptake inhibitors F) Tricyclic antidepressants

https://www.starttest.com/api/5.1.1.0/TTDStart.aspx?SVC=lbfd4732-460a-454b-90b5-9985c0560007

Exam Section 4: Item 7 o f 50 H Mark

National Board o f Medical Examiners Comprehensive Basic Science Self-Assessment

Tim e Remaining: 4 hr 8 min 34 sec

7. A 79-year-old woman with a history of severe dementia, type 2 diabetes mellitus, and hypertension is brought to the emergency department by her son because of chest pain and agitation for 4 hours. She had smoked 2 packs of cigarettes daily until the age of 70 years, when she quit. The son is not aware of the patient's medication regimen. Her pulse is 120/min, respirations are 32/min, and blood pressure is 180/100 mm Hg. Crackles are heard in both lung bases on auscultation of the chest. Cardiac examination shows a systolic ejection murmur at the apex and a regular rhythm. An ECG shows ST-segment elevation in the anterolateral leads. A chest x-ray shows a mildly enlarged cardiac silhouette. Which of the following is the most likely diagnosis?

= > A) Acute coronary syndrome o B) Acute pericarditis C) Bilateral pneumonia C D) Cerebrovascular event 0 E) Pulmonary embolism

National Board of Medical Examiners - Google Chrom e Q https://www.starttest.com/api/5.1.1.0/ITDStart.aspx?SVC=lbfd4732-460a-454b-90b5-9985c0560007

I__ I
National Board o f Medical Examiners Comprehensive Basic Science Self-Assessment Tim e Remaining: 4 hr 7 min 54 sec

Exam Section 4: Item 8 o f 50 I Mark

8. A 27-year-old woman comes to the physician because of a 5-day history of a moderate vaginal discharge. Physical examination shows a thin, white, vaginal discharge that has a foul smelling fishy odor. Vaginal pH is 6. A Pap smear shows clumps of bacteria on the squamous cells. There are no leukocytes. Culture of the discharge is most likely to show an increased amount of which of the following? A) Candida albicans B) Chlamydia trachomatis

C) Gardnerella vaginalis
D) Mycoplasma genitalium E) Trichomonas vaginalis

National Board of Medical Examiners - Google Chrom e Q https://www.starttest.com/api/5.1.1.0/ITDStart.aspx?SVC=lbfd4732-460a-454b-90b5-9985c0560007

I__ I
National Board o f Medical Examiners Comprehensive Basic Science Self-Assessment Tim e Remaining: 4 hr 6 min 38 sec

Exam Section 4: Item 9 o f 50 I Mark

9. A 35-year-old man who is an undocumented immigrant is diagnosed with tuberculosis. Despite feeling ill, he did not previously seek medical care because he could not afford to lose time at work. He pleads with the physician not to report the diagnosis to anyone because he is afraid he will be deported. After empathizing with the patient, it is most appropriate for the physician to take which of the following actions? A) Do not report the case and insist that the patient wear a mask at all times B) Do not report the case as long as the patient agrees to isolate himself while being treated C) Do not report the case because that would violate the patient's right to privacy and confidentiality D) Do not report the case, but begin contacting the patient's family members and work site contacts directly to arrange for evaluation E) Report the case to the health department but assure the patient that no other individuals will be notified o F) Report the case to the health department to ensure that family members and work site contacts are identified and evaluated G) Report the case to the immigration and naturalization service

National Board of Medical Examiners - Google Chrom e Q https://www.starttest.com/api/5.1.1.0/ITDStart.aspx?SVC=lbfd4732-460a-454b-90b5-9985c0560007

I__ I
National Board o f Medical Examiners Comprehensive Basic Science Self-Assessment Tim e Remaining: 4 hr 5 min 55 sec

Exam Section 4: Item 10 o f 50 I Mark

10.

An outbreak of multidrug-resistant Escherichia coli in an intensive care unit is being investigated. The determinants of antibiotic resistance are thought to be carried on a plasmid that has been transferred among different bacterial strains. Which of the following observations from in vitro studies best supports this hypothesis?

O A) Lysogeny must precede transfer B) Transfer is susceptible to DNase C) Transfer requires a bacteriophage o D) Transfer requires cell-to-cell contact E)Transfer requires transformation competent recipient strain

National Board of Medical Examiners - Google Chrom e Q https://wvw.starttest.com /api/511M TDStart.aspx7SVC3lbfd4732-460a-454b-90b5-9985c0560007

I___ I
National Board o f Medical Examiners Comprehensive Basic Science Self-Assessment Tim e Remaining: 4 hr 0 min 45 sec

Exam Section 4: Item 11 o f 50 0 Mark

11. A 45-year-old woman comes to the physician because of a 3-month history of decreased energy and impaired concentration. She has had tingling in both hands, constipation, and a 2-kg (4-lb) weight gain over the past 2 months. She appears apathetic. Her skin has a doughy consistency and her hair is coarse. Tapping the median nerves over the volar surface of the wrist elicits paresthesias in the index and middle fingers. The relaxation phase of the muscle stretch reflexes is slowed. Which of the following is the most likely diagnosis? O A) Addison disease B) Hyperparathyroidism C) Hypoparathyroidism D) Hypopituitarism o E) Hypothyroidism

National Board of Medical Examiners - Google Chrom e Q https: 7 www.starttest.com/api/5.1.1.0/ITDStart.aspx?SVC=lbfd4732-460a-454b-90b5-9985c0560007

I___ I
National Board o f Medical Examiners Comprehensive Basic Science Self-Assessment Tim e Remaining: 3 hr 59 min 19 sec

Exam Section 4: Item 12 o f 50 I Mark

12. In a clinical trial, a 44-year-old woman with end-stage renal disease undergoes kidney transplantation in combination with bone marrow transplantation from the same donor. Monitoring of the allograft post-transplantation shows good function despite the lack of treatment with immunosuppressive drugs. The transplanted bone marrow is most likely inducing which of the following immune phenomena in this recipient? O A) Co-stimulation B) Hypersensitivity O C) Senescence O D) Surveillance o E) Tolerance

National Board of Medical Examiners - Google Chrom e Q https://www.starttest.com/api/5.1.1.0/ITDStart.aspx?SVC=lbfd4732-460a-454b-90b5-9985c0560007

Exam Section 4: Item 13 o f 50 I Mark

National Board o f Medical Examiners Comprehensive Basic Science Self-Assessment

Tim e Remaining: 3 hr 53 min 49 sec

13. A 20-year-old man is brought to the emergency department 4 hours after receiving a gunshot wound to his forearm during a hunting accident. An exploratory operation shows that the tendon of the flexor carpi radialis is severed, and there is a comminuted fracture of the distal radius; all other structures are intact. Due to the extent of the injury, an attempt to suture the severed ends of the carpi radialis tendon is unsuccessful. The tendon of which of the following muscles is most likely to be used to surgically bridge the gap between the severed ends of the injured tendon because functional loss would be insignificant? A) Flexor carpi ulnaris C B) Flexor digitorum profundus C) Flexor digitorum s u p e rfic ia l o E) Pronator teres

C f e D) Palmaris longus

National Board of Medical Examiners - Google Chrom e Q https://www.starttest.com/api/5.1.1.0/ITDStart.aspx?SVC=lbfd4732-460a-454b-90b5-9985c0560007

I__ I
National Board o f Medical Examiners Comprehensive Basic Science Self-Assessment Tim e Remaining: 3 hr 52 min 45 sec

Exam Section 4: Item 14 o f 50 I Mark

14.

A cough suppressant (Drug X) is compared with codeine with the following results:

Characteristic
Cough suppression Analgesia Constipation Abuse potential

Codeine
++++ +++ +++ ++

Drug X
+++ minimal minimal +

Drug X most closely resembles which of the following drugs? A) Chlorpheniramine

o B) Dextromethorphan C) Guaifenesin D) Meperidine O E) Phenylephrine

National Board of Medical Examiners - Google Chrom e Q https://www.starttest.com/api/5.1.1.0/ITDStart.aspx?SVC=lbfd4732-460a-454b-90b5-9985c0560007

I__ I
National Board o f Medical Examiners Comprehensive Basic Science Self-Assessment Tim e Remaining: 3 hr 51 min 49 sec

Exam Section 4: Item 15 o f 50 I Mark

15. An investigative team is studying the physiologic mechanisms that explain increased exercise performance during 2 weeks at high altitude (10,000 feet above sea level) in experimental animals. These experiments are designed to distinguish between increases in arterial oxygen delivery to exercising muscle and unloading oxygen from blood into the tissues. Which of the following factors would be expected to increase for the data to support increased unloading oxygen into the tissues with acclimatization to high altitude? A) Arterial pH

B) Arterial Po2 o C) 2,3-Bisphosphoglycerate

0 D) Hematocrit 0 E) Net hemoglobin

L^p National Board of Medical Examiners - Google Chrom e

https://www.starttest.com/api/5.1.1.0/ITDStart.aspx?SVC=lbfd4732-460a-454b-90b5-9985c0560007

Exam Section 4: Item 16 o f 50 H Mark

National Board o f Medical Examiners Comprehensive Basic Science Self-Assessment

Tim e Remaining: 3 hr 50 min 34 sec

16. A 32-year-old woman with restless legs syndrome comes to the physician for a follow-up examination. She says that she now has symptoms three to four times weekly. A drug is prescribed that acts as an agonist on dopamine (D2) receptors. This drug is most likely which of the following? O A) Baclofen O B)Gabapentin O C)Levodopa o D) Ropinirole 0; E) Selegiline

Previous

Lab Values

Review

Pause
7:15 PM 04/08/2012

[^p National Board of Medical Examiners - Google Chrom e

I'

https://www.starttest.com/api/5.1.1.0/TTDStart.aspx?SVC=lbfd4732-460a-454b-90b5-9985c0560007

Exam Section 4: Item 17 o f 50 0 Mark

National Board o f Medical Examiners Comprehensive Basic Science Self-Assessment

Tim e Remaining: 3 hr 48 min 32 sec

17. A previously healthy 6-month-old boy develops fever and severe paroxysmal barking coughing. He has not had any immunizations. His leukocyte count is 30,000/mm3 (70% lymphocytes). Neutrophil chemotaxis and oxidative metabolism are defective due to increased activity of which of the following enzymes? o A) Adenylyl cyclase B) Myeloperoxidase

0 C) NADPH oxidase 0 D) Phospholipase C 0 E) Protein kinase C

Previous

O &
Next

Lab Values

Review

Help

Pause

H r% r i TO 1

..III

$ i >

7:17 PM
04/08/2012

National Board of Medical Examiners - Google Chrom e Q https://www.starttest.com/api/5.1.1.0/ITDStart.aspx?SVC=lbfd4732-460a-454b-90b5-9985c0560007

I___ I
National Board o f Medical Examiners Comprehensive Basic Science Self-Assessment Tim e Remaining: 3 hr 46 min 34 sec

Exam Section 4: Item 18 o f 50 0 Mark

18.

During a study of the response of renal tubular cells to ischemic injury, the blood supply to the kidneys in experimental animals is interrupted for variable lengths of time. The renal tubular cells then are examined by light and electron microscopy. The presence of which of the following features would definitively indicate irreversible injury to the renal tubular cells? A) Blunting of microvilli B) Cellular swelling C) Disaggregation of ribosomes

o D) Disruption of the plasma membrane E) Loosening of intercellular attachments

National Board of Medical Examiners - Google Chrom e Q https://www.starttest.com/api/5.1.1.0/ITDStart.aspx?SVC=lbfd4732-460a-454b-90b5-9985c0560007

I__ I
National Board o f Medical Examiners Comprehensive Basic Science Self-Assessment Tim e Remaining: 3 hr 45 min 11 sec

Exam Section 4: Item 19 o f 50 I Mark

19. A 4-year-old girl is brought to the emergency department 30 minutes after sustaining a burn on her right index finger when she touched a hot stove. Examination of the finger shows a tense, clear-fluid-filled blister. Before the blister resolves, epithelium forms at the base of the blister. Which of the following cells is involved in the re-epithelization of the blister in this patient? A) Basal layer keratinocytes C) Epithelial Langerhans cells

c >

C B) Dermal dendritic cells o D) Fibroblasts 0 E) Macrophages

National Board of Medical Examiners - Google Chrom e

4
National Board o f Medical Examiners Comprehensive Basic Science Self-Assessment Tim e Remaining: 3 hr 44 min 5 sec

h ttp s://w w w .sta rtte st.e o m /a p i/5 .l.1.0/TTDStart.aspx?SVC=lbfd4732-460a-454b-90b5-9985c0560007

Exam Section 4: Item 20 o f 50 I Mark

20.

A female newborn dies at the age of 12 days. A photograph of a section of the brain as seen at autopsy is shown. The newborn was most likely affected by which of the following? A) Arteriovenous malformation B) Germinal matrix hemorrhage

o C) Hyperbilirubinemia D) Hypoxia-ischemia E) Transtentorial herniation

Previous

O &
Next

Lab Values

Review

Help

Pause

H r% r i TO 1

..hi

$ i >

7:21 PM
04/08/2012

National Board of Medical Examiners - Google Chrom e Q https://www.starttest.com/api/5.1.1.0/ITDStart.aspx?SVC=lbfd4732-460a-454b-90b5-9985c0560007

Exam Section 4: Item 21 o f 50 I Mark

National Board o f Medical Examiners Comprehensive Basic Science Self-Assessment

Tim e Remaining: 3 hr 43 min 37 sec

21. An obese 45-year-old woman has a 16-year history of intermittent episodes of an epigastric burning sensation. Use of antacids usually relieves her symptoms. Esophagogastroscopy shows velvety patches in the mucosa of the distal esophagus. Which of the following best explains the process shown in the photomicrograph of a biopsy specimen from one of the patches?
A) Atrophy B) Hyperplasia O C) Hypertrophy o D) Metaplasia E) Steatosis

National Board of Medical Examiners - Google Chrom e Q https://www.starttest.com/api/5.1.1.0/ITDStart.aspx?SVC=lbfd4732-460a-454b-90b5-9985c0560007

I___ I
National Board o f Medical Examiners Comprehensive Basic Science Self-Assessment Tim e Remaining: 3 hr 40 min 40 sec

Exam Section 4: Item 22 o f 50 I Mark

22.

A 28-year-old man is brought to the emergency department because of blurred vision, fatigue, and severe thirst 20 minutes after being found wandering along a deserted farming road. He reports that his car had run out of gas 3 hours ago, and he had walked approximately 7 miles toward his destination without water or shade from the intense summer heat. His current body weight is 92% of the value listed on his driver's license. His temperature is 39.9C (103.8F), pulse is 105/min, and blood pressure is 95/60 mm Hg. Physical examination shows pale, dry skin. He is occasionally incoherent. He is placed supine on a bed, and intravenous fluids are administered. Within 60 minutes, the patient is able to take fluids by mouth and provides additional information about his next of kin before falling asleep. At the time when the man was found on the road, which of the following physiologic parameters would have been decreased compared with normal values? A) Plasma renin activity B) Renal (3-adrenergic receptor stimulation

o C) Renal arteriolar perfusion pressure D) Renal sympathetic nerve activation E) Serum angiotensin II concentration

National Board of Medical Examiners - Google Chrom e Q https://www.starttest.com/api/5.1.1.0/ITDStart.aspx?SVC=lbfd4732-460a-454b-90b5-9985c0560007

I___ I
National Board o f Medical Examiners Comprehensive Basic Science Self-Assessment Tim e Remaining: 3 hr 38 min 13 sec

Exam Section 4: Item 23 o f 50 I Mark

23.

Six weeks after undergoing an open carpal tunnel release operation, a 44-year-old woman comes to the physician because of new symptoms of numbness of her right hand. When she is asked about the exact location of these symptoms, she points to the area over the right thenar eminence. Percussion of the area between the flexor carpi radialis and palmaris longus tendons at the distal palmar wrist crease produces a painful shock-like sensation radiating into the affected area of the palm. An intraoperative nerve injury is suspected. Which of the following nerves is most likely injured in this patient? A) Dorsal sensory branch of the ulnar nerve B) Lateral cutaneous nerve of the forearm

C) Palmar cutaneous branch of the median nerve o D) Recurrent motor branch of the median nerve E) Sensory branch of the radial nerve

National Board of Medical Examiners - Google Chrom e Q https://www.starttest.com/api/5.1.1.0/ITDStart.aspx?SVC=lbfd4732-460a-454b-90b5-9985c0560007

I___ I
National Board o f Medical Examiners Comprehensive Basic Science Self-Assessment Tim e Remaining: 3 hr 36 min 19 sec

Exam Section 4: Item 24 o f 50 I Mark

24. An 18-year-old man develops a fever 6 days after returning to Wisconsin from a trip to India. He did not take antimalarial prophylaxis. A peripheral blood smear shows numerous intraerythrocytic forms of Plasmodium vivax. He is treated with chloroquine, and his symptoms resolve. Six months later, he again develops fever, and his blood smear again shows intraerythrocytic forms of P. vivax. He has not traveled since his trip to India. Which of the following is the most likely explanation for the recurrence of symptoms in this patient? o A) Activation of exoerythrocytic forms O B) Dual infection with P. falciparum C) Poor compliance with antimicrobial therapy O D) Reinfection O E) Resistance to chloroquine

National Board of Medical Examiners - Google Chrom e Q https://www.starttest.com/api/5.1.1.0/ITDStart.aspx?SVC=lbfd4732-460a-454b-90b5-9985c0560007

I___ I
National Board o f Medical Examiners Comprehensive Basic Science Self-Assessment Tim e Remaining: 3 hr 33 min 57 sec

Exam Section 4: Item 25 o f 50 0 Mark

25.

A 35-year-old man comes to the physician for a follow-up examination. He has a 10-year history of bradykinesia, tremors, and rigidity consistent with Parkinson disease. His parents are in their late 50s and have no symptoms of Parkinson disease. He has no siblings. Genetic analysis shows two loss-of-function mutations in the parkin gene, which encodes a ubiquitin protein ligase. Which of the following pathologic processes is most likely to occur in this patient as a result of these mutations? A) Absence of neuronal proteasomes

o B) Accumulation of aggregated protein within the cytoplasm of neurons C) Excess dopaminergic neurons in basal ganglia D) Increased protein aggregates within the nucleus of neurons E) Proliferation of pigmented neurons in the substantia nigra

National Board of Medical Examiners - Google Chrom e Q https://www.starttest.com/api/5.1.1.0/ITDStart.aspx?SVC=lbfd4732-460a-454b-90b5-9985c0560007

I___ I
National Board o f Medical Examiners Comprehensive Basic Science Self-Assessment Tim e Remaining: 3 hr 30 min 6 sec

Exam Section 4: Item 26 o f 50 I Mark

26. A 42-year-old woman comes for a routine follow-up examination. She has an 8-year history of hypertension treated with a thiazide diuretic. She reports that she tires easily with exertion because of her weight but is otherwise asymptomatic. She drinks one to two glasses of wine weekly. There is no family history of liver disease. She is 163 cm (5 ft 4 in) tall and weighs 77 kg (170 lb); BMI is 29 kg/m2. Her blood pressure is 140/90 mm Hg. Examination shows no other abnormalities except for truncal obesity with no abdominal striae. Serum studies show: Urea nitrogen Glucose Creatinine Total cholesterol HDL-cholesterol Triglycerides AST ALT 19mg/dL 117mg/dL 1 mg/dL 227 mg/dL 32 mg/dL 347 mg/dL 87 U/L 85 U/L

Which of the following is the most likely cause of this patient's increased serum liver enzyme activity? O A) Alcoholic hepatitis B) Hemochromatosis C) Hepatitis A o D) Nonalcoholic steatohepatitis E) Porphyria cutanea tarda

f Q

R a tio n a l Board of Medical Examiners - Google Chrom e https://www.starttest.eom/api/5.1.1.0/TTDStart.aspx?SVC=lbfd4732-460a-454b-90b5-9985c0560007

Exam Section 4: Item 27 o f 50 I Mark

National Board o f Medical Examiners Comprehensive Basic Science Self-Assessment

Tim e Remaining: 3 hr 29 min 22 sec

27.

A 25-year-old man who is seropositive for HIV has had progressive apathy and dysarthria for 3 months. Examination shows a homonymous hemianopia. An MRI shows multiple enhancing lesions in the cerebral cortex. Which of the following is the most likely diagnosis?

A) Colloid cyst B) Craniopharyngioma O C) Ependymoma D) Ganglioneuroma o E) Lymphoma F) Medulloblastoma G) Meningioma H) Retinoblastoma I) Schwannoma

National Board of Medical Examiners - Google Chrom e Q https://www.starttest.com/api/5.1.1.0/ITDStart.aspx?SVC=lbfd4732-460a-454b-90b5-9985c0560007

I___ I
National Board o f Medical Examiners Comprehensive Basic Science Self-Assessment Tim e Remaining: 3 hr 27 min 57 sec

Exam Section 4: Item 28 o f 50 I Mark

28. A 32-year-old African American man comes to the physician for a routine health maintenance examination. He feels well and has no history of serious illness. His brother received the diagnosis of anemia at the age of 20 years. The patient's pulse is 76/min, respirations are 16/min, and blood pressure is 128/68 mm Hg. Examination shows no abnormalities. Laboratory studies show: Hemoglobin Mean corpuscular volume Red cell distribution width Serum Ferritin Iron 11 g/dL 70 pm3 13% (N=13% -15%) 200 ng/mL 150 pg/dL

A blood smear shows hypochromic microcytic erythrocytes with occasional target cells. Which of the following is most likely to confirm the diagnosis? o A) Hemoglobin electrophoresis B) Measurement of serum thyroid-stimulating hormone concentration C) Measurement of serum vitamin B 12 (cobalamin) concentration O D) Colonoscopy O E) Bone marrow biopsy

[^p National Board of Medical Examiners - Google Chrom e

I'

h ttp s://w w w .sta rtte st.e o m /a p i/5 .l.1.0/TTDStart.aspx?SVC=lbfd4732-460a-454b-90b5-9985c0560007

Exam Section 4: Item 29 o f 50 I Mark

National Board o f Medical Examiners Comprehensive Basic Science Self-Assessment

Tim e Remaining: 3 hr 26 min 45 sec

29. An investigator conducts a study of gastrointestinal drugs that can be used preoperatively to minimize the risk for regurgitation and pulmonary aspiration of gastric contents during an operation. It is found that one of these drugs promotes gastric emptying and relieves nausea and vomiting. This drug is most likely which of the following? A) Dronabinol

o B) Metoclopramide O C) Misoprostol 0 D) Ondansetron 0 E) Scopolamine

Previous

O &
Next

Lab Values

Review

Help -

Pause ..in $ < > 7:39 PM


04/08/2012

H r% r i TO 1

National Board of Medical Examiners - Google Chrom e Q https://www.starttest.com/api/5.1.1.0/ITDStart.aspx?SVC=lbfd4732-460a-454b-90b5-9985c0560007

I__ I
National Board o f Medical Examiners Comprehensive Basic Science Self-Assessment Tim e Remaining: 3 hr 24 min 40 sec

Exam Section 4: Item 30 o f 50 I Mark

30. A study is conducted to assess whether a new antiviral drug is effective in treating symptomatic outbreaks in patients with herpes simplex virus (HSV) infections. A total of 1000 patients from a community with recurrent HSV infections are enrolled and randomized: 500 patients receive the new drug, and 500 patients receive placebo. Results show that the average duration of recurrent herpes simplex outbreaks in the new drug group is 1 week, compared with 2 weeks in the placebo group. Which of the following outcomes regarding symptomatic HSV infections is most likely in the greater population if all affected patients in the community are treated with the new drug? O A) Incidence will decrease by half O B) Incidence will double o C) Prevalence will decrease by half O D) Prevalence will double

National Board of Medical Examiners - Google Chrom e Q https://www.starttest.com/api/5.1.1.0/ITDStart.aspx?SVC=lbfd4732-460a-454b-90b5-9985c0560007

Exam Section 4: Item 31 o f 50 I Mark

National Board o f Medical Examiners Comprehensive Basic Science Self-Assessment

Tim e Remaining: 3 hr 23 min 0 sec

31.

During an experiment, the US11 gene product of cytomegalovirus is expressed constitutively after stable transfection in an experimental tumor cell line. It is found that this gene product causes translocation of nascent class I MHC molecules from the endoplasmic reticulum into the cytosol. Which of the following is most likely to occur regarding the class I MHC products? A) Association with invariant chain

C B) Binding of peptides from the endocytic pathway = > C) Degradation by the proteasome D) Formation of class I MHC/class II MHC hybrid molecules o E) Interaction with the T-lymphocyte receptor

[^p National Board of Medical Examiners - Google Chrom e

I'

https://www.starttest.com/api/5.1.1.0/TTDStart.aspx?SVC=lbfd4732-460a-454b-90b5-9985c0560007

Exam Section 4: Item 32 o f 50 I Mark

National Board o f Medical Examiners Comprehensive Basic Science Self-Assessment

Tim e Remaining: 3 hr 21 min 45 sec

32. A 58-year-old man who has smoked 2 packs of cigarettes daily for 30 years has severe chronic bronchitis. Which of the following arterial blood gas values with the patient breathing room air are most compatible with his condition? pH A) B) C) D) E) 7.30 7.32 7.42 7.50 7.52 P c o 2(mm Hg) 18 65 40 55 20 P o 2(mm 95 60 95 90 60

Previous

Lab Values

Review

Help

Pause

O
.,il, $ < >
7:44 PM 04/08/2012

National Board of Medical Examiners - Google Chrom e Q https://www.starttest.com/api/5.1.1.0/ITDStart.aspx?SVC=lbfd4732-460a-454b-90b5-9985c0560007

Exam Section 4: Item 33 o f 50 0 Mark

National Board o f Medical Examiners Comprehensive Basic Science Self-Assessment

Tim e Remaining: 3 hr 20 min 6 sec

33. During a study, a healthy 35-year-old woman and a 35-year-old woman with Crohn disease are given an oral solution of radiolabeled lactulose. The appearance of radioactivity in their urine is monitored over the next 24 hours. Urinary excretion of radioactivity in the patient with Crohn disease is much greater than that of the healthy subject. This finding is most likely due to inflammation-associated changes in the integrity of which of the following epithelial structures? o A) Adherens junctions (zonulae adherentes) C B) Basement membrane C) Desmosomes (maculae adherentes) D) Gap junctions = > E) Tight junctions (zonulae occludentes)

National Board of Medical Examiners - Google Chrom e Q https://www.starttest.com/api/5.1.1.0/ITDStart.aspx?SVC=lbfd4732-460a-454b-90b5-9985c0560007

I___ I
National Board o f Medical Examiners Comprehensive Basic Science Self-Assessment Tim e Remaining: 3 hr 19 min 0 sec

Exam Section 4: Item 34 o f 50 I Mark

34.

A 40-year-old man comes to the physician for a routine health maintenance examination. There is a strong family history of myocardial infarctions occurring at the ages of 40 to 50 years. Physical examination shows no abnormalities. Serum studies show: Cholesterol, total HDL-cholesterol LDL-cholesterol Triglycerides increased normal increased normal

Following treatment with lovastatin, his serum total cholesterol and serum LDL-cholesterol concentrations decrease. The beneficial effect of lovastatin most likely occurs as a result of which of the following mechanisms? A) Decreases absorption of bile acids in intestine B) Increases hydrolysis of triglycerides o C) Increases LDL-receptors D) Increases lipoprotein lipase activity O E) Inhibits the secretion of apo B-48

[^f National Board of Medical Examiners - Google Chrom e

Q https://www.starttest.com/api/5.1.1.0/TTDStart.aspx?SVC=lbfd4732-460a-454b-90b5-9985c0560007 Exam Section 4: Item 35 o f 50 H Mark National Board o f Medical Examiners Comprehensive Basic Science Self-Assessment Tim e Remaining: 3 hr 17 min 6 sec

35. A lesion of which of the following lettered structures results in loss of the pupillary light reflex in the left eye when light is shone into either eye?

Previous

Next

Lab Values

Review

Pause

C l
7:49 PM 04/08/2012

National Board of Medical Examiners - Google Chrom e Q https://www.starttest.com/api/5.1.1.0/ITDStart.aspx?SVC=lbfd4732-460a-454b-90b5-9985c0560007

I__ I
National Board o f Medical Examiners Comprehensive Basic Science Self-Assessment Tim e Remaining: 3 hr 13 min 35 sec

Exam Section 4: Item 36 o f 50 I Mark

36.

After an emergency landing in the wilderness, a 25-year-old pilot goes 3 days without food until she is rescued. Adipocytes play an important role in maintaining homeostasis in this woman because of which of the following effects? A) Glucagon activates glycerol utilization as a source of carbon for gluconeogenesis by adipocytes B) Glucagon activates glycogen mobilization to form free glucose

o C) Glucagon activates hormone-sensitive lipase D) Insulin activates fatty acid synthesis E) Insulin activates hepatic lipoprotein lipase

National Board of Medical Examiners - Google Chrom e Q https://www.starttest.com/api/5.1.1.0/ITDStart.aspx?SVC=lbfd4732-460a-454b-90b5-9985c0560007

I__ I
National Board o f Medical Examiners Comprehensive Basic Science Self-Assessment Tim e Remaining: 3 hr 10 min 54 sec

Exam Section 4: Item 37 o f 50 0 Mark

37.

A 16-year-old boy who frequently goes fishing is brought to the physician because of a 1-month history of a lesion on the front of his right shoulder. He has blond hair and fair skin. Physical examination shows an 0.8-cm ulcerated, slightly raised nodule that is variably brown to black. Examination of a biopsy specimen of the nodule confirms a superficially invasive melanoma with features of regression. Which of the following best explains the lesion's regression in this patient? A) Antibody-dependent cellular reaction

o B) Antibody-mediated cellular dysfunction O C) Arthus reaction D) Complement- and Fc receptor-mediated response E) T-lymphocyte-mediated cytotoxicity

National Board of Medical Examiners - Google Chrom e Q https://www.starttest.com/api/5.1.1.0/ITDStart.aspx?SVC=lbfd4732-460a-454b-90b5-9985c0560007

I___ I
National Board o f Medical Examiners Comprehensive Basic Science Self-Assessment Tim e Remaining: 3 hr 9 min 38 sec

Exam Section 4: Item 38 o f 50 I Mark

38.

A 38-year-old man comes to the physician because of a 3-week history of rectal pain and bleeding that is worse when he defecates. He had similar symptoms 1 year ago. He is afebrile. Anoscopy shows the findings in the photograph. This patient most likely has which of the following conditions?

O A) Anal fistula B) Colorectal carcinoma C) Condylomata acuminata o D) Hemorrhoids O E) Perianal abscess

National Board of Medical Examiners - Google Chrom e Q https://www.starttest.com/api/5.1.1.0/ITDStart.aspx?SVC=lbfd4732-460a-454b-90b5-9985c0560007

I___ I
National Board o f Medical Examiners Comprehensive Basic Science Self-Assessment Tim e Remaining: 3 hr 6 min 44 sec

Exam Section 4: Item 39 o f 50 I Mark

39.

A 34-year-old man with an 8-year history of HIV infection comes to the physician for a follow-up examination. His condition has remained stable with highly active antiretroviral therapy consisting of one nonnucleoside reverse transcriptase inhibitor (NNRTI) and two nucleoside reverse transcriptase inhibitors (NRTIS). Blockade of which of the following best explains the beneficial effects of NNRTI therapy in this patient?

O A) Apoptosis of HIV-infected cells B) D) Cleavage of viral envelope precursor glycoproteins Integrase functions in host cells O C) Entry of HIV virus into host T lymphocytes o E) Replication of the HIV viral genome in host cells

https://www.starttest.com/api/5.1.1.0/TTDStart.aspx?SVC=lbfd4732-460a-454b-90b5-9985c0560007

Exam Section 4: Item 40 o f 50 H Mark

National Board o f Medical Examiners Comprehensive Basic Science Self-Assessment

Tim e Remaining: 3 hr 5 min 26 sec

40. A 50-year-old man comes to the physician for a follow-up examination. He has poorly controlled hypertension despite three-drug therapy. He is 183 cm (6 ft) tall and weighs 90 kg (198 lb); BMI is 27 kg/m2. His pulse is 72/min and regular, and blood pressure is 180/102 mm Hg. An S 4and an abdominal bruit are heard on auscultation. Serum studies show a sodium concentration of 142 mEq/L, potassium concentration of 2.4 mEq/L, and creatinine concentration of 1 mg/dl_. MR angiography of the abdomen shows narrowing of the right renal artery. Which of the following sets of hormone findings is most likely in this patient?

Plasma Renin
m
A) B) C) D) E) F) G) H) I) * r Normal Normal Normal

Serum Aldosterone
* normal r normal

normal

National Board of Medical Examiners - Google Chrom e Q https://www.starttest.com/api/5.1.1.0/ITDStart.aspx?SVC=lbfd4732-460a-454b-90b5-9985c0560007

I__ I
National Board o f Medical Examiners Comprehensive Basic Science Self-Assessment Tim e Remaining: 3 hr 3 min 47 sec

Exam Section 4: Item 41 o f 50 I Mark

41. A 35-year-old woman with leiomyomata uteri undergoes an experimental treatment that involves instillation of an embolizing agent directly into the vessels that feed the leiomyomata. During this procedure, in order to reach the involved vessels, which of the following is the most direct course of the catheter after entering the femoral artery? A) Aorta -> gonadal artery -> uterine artery o B) External iliac artery -> internal iliac artery -> uterine artery O C) External iliac artery -> superior gluteal artery -> uterine artery O D) Internal iliac artery -> inferior vesical artery -> uterine artery O E) Internal iliac artery -> superior vesical artery -> uterine artery

National Board of Medical Examiners - Google Chrom e Q https://www.starttest.com/api/5.1.1.0/ITDStart.aspx?SVC=lbfd4732-460a-454b-90b5-9985c0560007

I___ I
National Board o f Medical Examiners Comprehensive Basic Science Self-Assessment Tim e Remaining: 3 hr 2 min 48 sec

Exam Section 4: Item 42 o f 50 I Mark

42. A 40-year-old man with type 2 diabetes mellitus comes to the physician for a follow-up examination. He currently takes no medications, and his condition is being treated by diet. He has a sedentary lifestyle. He is 178 cm (5 ft 10 in) tall and weighs 104 kg (230 lb); BMI is33 kg/m2.Physical examination shows no other abnormalities. His fasting serum hemoglobin A 1c is 9%. Treatment with glyburide is started. This drug is most likely to have a beneficial effect in thispatient by whichof the following mechanisms? o A) Blocking an ATP-sensitive potassium channel in the pancreatic beta cells B) Blocking the conversion of sucrose to glucose in the liver C) Inhibiting a cytochrome P450 enzyme that degrades insulin D) Interfering with the absorption of glucose from the gastrointestinal tract E) Stimulating insulin uptake by the liver by promoting the formation of caveolae in the membrane

National Board of Medical Examiners - Google Chrom e

h ttp s://w w w .sta rtte st.e o m /a p i/5 .l.1.0/TTDStart.aspx?SVC=lbfd4732-460a-454b-90b5-9985c0560007

Exam Section 4: Item 43 o f 50 I Mark

National Board o f Medical Examiners Comprehensive Basic Science Self-Assessment

Tim e Remaining: 3 hr 1 min 57 sec

>y %. '4

*f
. 'v #
. < *# I* .

, * * *> <
i r

, . . . i

v * * .

' K

V ?

43. A 24-year-old woman comes to the physician because of a 2-week history of hoarseness and difficulty swallowing. She has a 14-year history of mild persistent asthma successfully treated with daily administration of inhalational fluticasone and albuterol as needed. A photograph of the tongue is shown. Microscopic examination of scrapings from these areas shows budding yeast. Which of the following is the most appropriate pharmacotherapy for this patient's oral infection? A) Doxycycline

O B) Flucytosine o C) Nystatin O D) Penicillin G benzathine E)Terbinafine

https://www.starttest.com/api/5.1.1.0/TTDStart.aspx?SVC=lbfd4732-460a-454b-90b5-9985c0560007

Exam Section 4: Item 44 o f 50 H Mark

National Board o f Medical Examiners Comprehensive Basic Science Self-Assessment

Tim e Remaining: 3 hr 0 min 44 sec

44. A 60-year-old man comes to the physician for a routine health maintenance examination. Physical examination shows no abnormalities. Urinalysis shows: pH Specific gravity Blood Glucose Protein 6.0 1.018 3+ negative 1+

Microscopic examination of the urine shows atypical cells. A CT scan of the abdomen discloses a lesion of the right kidney as shown in the photograph. Which of the following compounds is the most significant predisposing risk factor for this patient's condition? A) Arsenic B) Beryllium o C) 2-Naphthylamine D) Nickel E) Vinyl chloride

National Board of Medical Examiners - Google Chrom e Q https://www.starttest.com/api/5.1.1.0/ITDStart.aspx?SVC=lbfd4732-460a-454b-90b5-9985c0560007

I__ I
National Board o f Medical Examiners Comprehensive Basic Science Self-Assessment Tim e Remaining: 2 hr 59 min 51 sec

Exam Section 4: Item 45 o f 50 I Mark

45. A 59-year-old man comes to the physician because of a 3-week history of dull left-sided chest pain, shortness of breath, weakness, and nonproductive cough. He also has had an 11 -kg (25-lb) weight loss during the past 3 months. He has worked as an insulation installer for 30 years. He is 185 cm (6 ft 1 in) tall and now weighs 70 kg (155 lb); BMI is 20 kg/m2. His respirations are 26/min. Breath sounds are diminished on the left side. A chest x-ray is shown. A biopsy specimen shows an anaplastic, biphasic neoplasm that expresses calretinin and cytokeratin but not carcinoembryonic antigen. Which of the following additional structures is most likely in this patient's lung tissue? O A) Birbeck granules B) Dense core secretory granules o C) Ferruginous bodies O D) Signet ring cells E) Silica crystals

' [^N ational Board of Medical Examiners - Google Chrom e

https://www.starttest.com/api/5.1.1.0/TTDStart.aspx?SVC=lbfd4732-460a-454b-90b5-9985c0560007

Exam Section 4: Item 46 o f 50 I Mark

National Board o f Medical Examiners Comprehensive Basic Science Self-Assessment

Tim e Remaining: 2 hr 57 min 52 sec

46.

An investigation is conducted to examine a new drug (Drug X) for the treatment of asthma. The drug is found to increase cAMP in smooth muscle cells. The mechanism of action of Drug X most closely resembles that of which of the following drugs?

o A) Albuterol B) Ipratropium C) Montelukast O D) Nedocromil O E)Omalizumab

Previous

Lab Values

Review

Help
-

Pause

O
.,il,

8:08 PM
< >

04/08/2012

National Board of Medical Examiners - Google Chrom e Q https://www.starttest.com/api/5.1.1.0/ITDStart.aspx?SVC=lbfd4732-460a-454b-90b5-9985c0560007

I__ I
National Board o f Medical Examiners Comprehensive Basic Science Self-Assessment Tim e Remaining: 2 hr 56 min 32 sec

Exam Section 4: Item 47 o f 50 I Mark

47. A 52-year-old man comes to the physician for a follow-up visit 2 weeks after being released from the hospital following a myocardial infarction. Since discharge, he has not had chest pain, dyspnea on exertion, paroxysmal nocturnal dyspnea, orthopnea, or edema. He has smoked 2 packs of cigarettes daily for the past 35 years, except for the week he was in the hospital. When the physician asks him about smoking cessation, he says he is not interested. Which of the following is the most appropriate next step to help the patient stop smoking? o A) Ask the patient if he has any concerns about smoking and his health O B) Give him a prescription for bupropion C) Give him a prescription for nicotine replacement therapy D) Have the patient set a quit date to stop smoking E) Tell the patient he will have another myocardial infarction if he continues to smoke F) Tell the patient he will need to find another physician if he does not quit smoking

National Board of Medical Examiners - Google Chrom e Q https://www.starttest.com/api/5.1.1.0/ITDStart.aspx?SVC=lbfd4732-460a-454b-90b5-9985c0560007

I__ I
National Board o f Medical Examiners Comprehensive Basic Science Self-Assessment Tim e Remaining: 2 hr 56 min 1 sec

Exam Section 4: Item 48 o f 50 I Mark

48.

A 67-year-old man comes to the physician because of tingling and numbness of his hands for 10 days. He has non-Hodgkin lymphoma treated with cyclophosphamide, doxorubicin, prednisone, rituximab, and vincristine. Sensation to pinprick is decreased over the hands and feet. Which of the following drugs in this patient's medication regimen is the most likely cause of these findings? A) Cyclophosphamide B) Doxorubicin

C) Prednisone D) Rituximab o E) Vincristine

National Board of Medical Examiners - Google Chrom e Q https://www.starttest.com/api/5.1.1.0/ITDStart.aspx?SVC=lbfd4732-460a-454b-90b5-9985c0560007

Exam Section 4: Item 49 o f 50 I Mark

National Board o f Medical Examiners Comprehensive Basic Science Self-Assessment

Tim e Remaining: 2 hr 54 min 56 sec

49.

A 60-year-old man develops pain, erythema, and swelling of the right great toe. Serum uric acid concentration is three times normal. Which of the following findings is most common in patients with this condition? A) Absence of aminotransferase B) Absence of glucose 6-phosphatase C) Absence of glutathione peroxidase

o D) Absence of hypoxanthine guanine phosphoribosyltransferase = > E) No specific enzyme or renal defect

You might also like